Sunteți pe pagina 1din 69

User Name : chandan paswan

Total Marks : 200


TEST - 16 Mark Scored : 104.67

1 Consider the following statements about the All-India trends of inflation.


1. Assertion (A): Urban inflation has always been more than rural Inflation in post-
independent India.
2. Reason (R): Urban India constitutes a larger share of India's GDP than rural areas.
In the context of the above, which of these is correct?
A. A is correct, and R is an appropriate explanation of A.
B. A is correct, but R is not an appropriate explanation of A.

S
C. A is correct, but R is incorrect.
D. A is incorrect, but R is correct.
User Answer : A

IA
Correct Answer : D
Answer Justification :

Justification: India's urban sector presently constitutes 63% of India's GDP (up
from 45% in 1990) and has been its key engine of growth acceleration in the past
TS
decade. So, R is correct.

If you observe the trends given in India Yearbook for the year 2013 (Chapter 6)
itself, the trends do not have a uniform pattern. It is more in rural sometimes, and
more in urban areas sometimes. A is incorrect.
H
Learning:With only 31% of India's population currently urbanized, along with high
population density and low GDP per capita, India's urbanization trends have scope
to significantly accelerate.
G

Q SourceChapter 6: India Yearbook 2016


SI

2 Remittances are an important source of financial flows for India. Which of the following
factors may lead to an increase in remittance inflows to India?
1. Higher oil prices in the Gulf countries
IN

2. Depreciation of Rupee
Which of the above is/are correct?
A. 1 only
B. 2 only
C. Both 1 and 2
D. None
User Answer : B
Correct Answer : C
Answer Justification :

Justification:Statement 1: Higher oil prices help the Gulf countries and other oil

(C) Insights Active Learning. | All rights reserved. www.insightsias.com 1


User Name : chandan paswan
Total Marks 200
TEST - 16 Mark Scored 104.67

exporters to earn higher profits/income (where a large number of Indian workers are
employed - whose higher earnings made higher transfers possible to India).So, 1 can
be correct.

Statement 2: Depreciation of rupee increases the value of foreign currency in India


and hence the NRIs or OCIs are at an advantage sending remittances to India.

Q Source:Chapter 10: Ramesh Singh: Indian Economy

S
3 Which of the following is/are the financial powers of the President of India?

IA
1. Money bills can be introduced in the Parliament only with his prior
recommendation.
2. He causes to be laid before the Parliament the Union Budget.
3. No demand for a grant can be made except on his recommendation.
4. He can make advances out of the contingency fund of India to meet any unforeseen
expenditure.
TS
5. He constitutes the Union finance commission to recommend the distribution of
revenues between the Centre and the states.
Select the correct answer using the codes below.
A. 3 and 5 only
H
B. 1, 2 and 5 only
C. 1, 2 and 4 only
D. 1, 2, 3, 4 and 5
G

User Answer : D
Correct Answer : D
Answer Justification :
SI

Justification: Statement 1 and 3: Both statements are correct. More on these lines
will be covered in the coming tests.
IN

Statement 2: President shall, in respect of every financial year, cause to be laid


before Parliament, Annual Financial Statement as per Article 112 of the
Constitution. The Budget is presented to Parliament on such date as is fixed by the
President.

Statement 4: The fund is held by the Finance Secretary (Department of Economic


Affairs) on behalf of the President of India and it can be operated by executive
action.

Q Source:Chapter 17: Indian Polity: M Laxmikanth

(C) Insights Active Learning. | All rights reserved. www.insightsias.com 2


User Name : chandan paswan
Total Marks : 200
TEST - 16 Mark Scored : 104.67

4 Ahmadabad Mill Strike 1918 led by Gandhiji was based on the issue of
A. Plague Bonus of the previous year to workers
B. Inhuman treatment of mill workers by the management
C. Objection of management on participation of workers in the freedom struggle
D. Large-scale layoffs of mill workers
User Answer :
Correct Answer : A

S
Answer Justification :

Learning:In 1918, there was a situation of conflict between the Gujarat Mill owners

IA
and workers on the question of Plague Bonus of 1917.

The Mill Owners wanted to withdraw the bonus whole the workers demanded a 50%
wage hike. The Mill Owners were willing to give only 20% wage hike.

In March later, under the leadership of Gandhi, there was a strike in the cotton mills.
TS
In this strike Gandhi used the weapon of Hunger strike.

It was carried out in pure non-violent disciplined way. The result was that the strike
was successful and the workers got a 35% wage increase.
H
Q Source: Chapter 12: Bipin Chandra: India's Struggle for Independence
G

5 Hannover Messe 2015 was a/an


A. Energy summit held under the aegis of Conference of Parties (COP) of
UNFCCC
SI

B. The cultural exchange programme between India and Europe organized by


the Ministry of Culture and Ministry of Tourism
C. An annual global exhibition held in Germany which provided a platform for
India to gain entry to international markets
IN

D. None of the above


User Answer :
Correct Answer : C
Answer Justification :

Learning:The Hannover Messe (HM) is one of the most important annual global
exhibitions which provided an ideal platform for India to gain entry to international
markets with focus on global technological and industrial innovation held in
Hanover, Germany in April, 2015. 'Make in India' was the theme of the Hanover
Fair.

(C) Insights Active Learning. | All rights reserved. www.insightsias.com 3


User Name : chandan paswan
Total Marks 200
TEST - 16 Mark Scored 104.67

As India was designated as a "Partner Country" by Hanover Messe, a rare


distinction achieved by any country for the second time in the last ten years, a high
profile platform for the presentation of India's industrial and business achievements
was geared up by this Department in close coordination with the DIPP.

The high level political and economic attendance at HM brings worldwide publicity.

S
6 When a vacancy occurs in the office of the President due to his resignation, the Vice-
President acts as the President until a new President is elected. Consider the following with
regard to when the office of Vice-President is vacant.

IA
1. Assertion (A): Speaker, Lok Sabha acts as the President of India in such a case.
2. Reason (R): Speaker, Lok Sabha is ranked immediately below the Vice-President
and Prime Minister in the Table of Precedence.
In the context of the above, which of these is correct?
A. A is correct, and R is an appropriate explanation of A.
TS
B. A is correct, but R is not an appropriate explanation of A.
C. A is incorrect, but R is correct.
D. Both A and R are incorrect.
User Answer : D
Correct Answer : D
H
Answer Justification :

Justification:When a vacancy occurs in the office of the President due to his


G

resignation, removal, death or otherwise, the Vice-President acts as the President


until a new President is elected.
SI

Further, when the sitting President is unable to discharge his functions due to
absence, illness or any other cause, the Vice-President discharges his functions until
the President resumes his office.
IN

In case the office of Vice-President is vacant, the Chief Justice of India (or if his
office is also vacant, the senior-most judge of the Supreme Court available) acts as
the President or discharges the functions of the President.

Q Source:Chapter 17: Indian Polity: M Laxmikanth

7 Which of the following statements about the Archaeological Survey of India (ASI) is
INCORRECT?
A. It was established before independence.
B. It functions as an attached office of the Department of Culture.

(C) Insights Active Learning. | All rights reserved. www.insightsias.com 4


User Name : chandan paswan
Total Marks : 200
TEST - 16 Mark Scored : 104.67

C. Expeditions abroad' is one of the major activities of the organization.


D. ASI cannot take up conservation projects of sites outside India.
User Answer : D
Correct Answer : D
Answer Justification :

Justification:The ASI has taken up the conservation project of Ta Prohm,

S
Cambodia under the ITEC programme of Ministry of External Affairs with an
outlay of ? 19.51 crore.

IA
The conservation project has been started on the request of the Royal Government of
Cambodia for India's assistance in Conservation and Restoration of Prasat Ta
Prohm.

The major activities of the Archaeological Survey of India are:


TS
Survey of archaeological remains and excavations;
Maintenance and conservation of Centrally protected monuments, sites and
remains;
Chemical preservation of monuments and antiquarian remains;
H
Architectural survey of monuments;

Q Source: Chapter 5: India Yearbook 2016


G

8 In case of a Parliamentary deadlock, the President summons a joint sitting of both houses
to resolve the issue. Who chairs the joint sitting?
SI

A. President
B. Chairman, Rajya Sabha
C. Speaker, Lok Sabha
IN

D. Leader of the House of People


User Answer : C
Correct Answer : C
Answer Justification :

Learning:The President is an integral part of the Parliament of India, and enjoys the
following legislative powers:

He can summon or prorogue the Parliament and dissolve the Lok Sabha.
He can address the Parliament at the commencement of the first session after

(C) Insights Active Learning. | All rights reserved. www.insightsias.com 5


User Name : chandan paswan
Total Marks 200
TEST - 16 Mark Scored 104.67

each general election and the first session of each year.


He can send messages to the Houses of Parliament, whether with respect to a
bill pending in the Parliament or otherwise.
He can appoint any member of the Lok Sabha to preside over its proceedings
when the offices of both the Speaker and the Deputy Speaker fall vacant.

Q Source:Chapter 17: Indian Polity: M Laxmikanth

S
9 Consider the following statements.
1. Assertion (A): All doubts and disputes in connection with election of the President

IA
are inquired into and decided by the Election Commission of India.
2. Reason (R): Election Commission of India is entrusted with the responsibility of
conducting the Presidential elections.
In the context of the above, which of these is correct?
A. A is correct, and R is an appropriate explanation of A.
TS
B. A is correct, but R is not an appropriate explanation of A.
C. A is correct, but R is incorrect.
D. A is incorrect, but R is correct.
User Answer : D
Correct Answer : D
H
Answer Justification :

Justification:Supreme Court decides all the doubts and disputes in connection with
G

election of the President even though ECI organizes and conducts the elections.

So, A is incorrect, and R is correct.


SI

Moreover, ECI has only limited quasi-judicial authority with regard to elections of
MPs or MLAs. We will be covering these topics in the coming tests.
IN

Q Source: Chapter 17: Indian Polity: M Laxmikanth

10 It is said that the US-India Defense Technology and Partnership Act that proposes to
amend the 'Arms Export Control Action' is of immense strategic significance to India.
What can be the possible reason(s)?
1. It seeks to formalise India's status as a major partner of equal status as US's NATO
allies and closest partners.
2. It will elevate India's status in export of defence articles from the United States to
India.
Which of the above is/are correct?

(C) Insights Active Learning. | All rights reserved. www.insightsias.com 6


User Name : chandan paswan
Total Marks : 200
TEST - 16 Mark Scored : 104.67

A. 1 only
B. 2 only
C. Both 1 and 2
D. None
User Answer : A
Correct Answer : C
Answer Justification :

S
Learning: Defence trade between the US and India is one of the strongest areas of
the bilateral economic relationship. Over the past 10 years it has risen from $300

IA
million to over $14 billion.

The act has been introduced by Congressman George Holding, Co-Chair of the
House India Caucus. It will help
TS
Cement the India-US relations and lay a foundation for future cooperation
and growth.
Shorten the time required for the notification of sale or export of defence
articles from the United States to India.
Encourage more joint contingency planning and require the US government
H
review and assess India's ability to execute military operations of mutual
interest.
G

Q
Source:http://economictimes.indiatimes.com/news/defence/resolution-introduced-to
-bring-india-on-par-with-nato-allies/articleshow/51527876.cms
SI

11 The Ghadar party was a revolutionary organization with the aim of securing India's
independence from British rule. Who among the following were associated with it?
IN

1. Lala Har Dayal


2. Rashbehari Bose
3. Sohan Singh Bhakna
Select the correct answer using the codes below.
A. 1 and 2 only
B. 2 and 3 only
C. 1 and 3 only
D. 1, 2 and 3
User Answer : C
Correct Answer : D
Answer Justification :

(C) Insights Active Learning. | All rights reserved. www.insightsias.com 7


User Name : chandan paswan
Total Marks 200
TEST - 16 Mark Scored 104.67

Learning:The Ghadar Party was an organization founded by Punjabi Indians in the


United States and Canada with the aim of securing India's independence from
British rule.

Key members included Lala Har Dayal, Sohan Singh Bhakna, Abdul Hafiz
Mohamed Barakatullah, Kartar Singh Sarabha, and Rashbehari Bose.

After the outbreak of World War I, Ghadar party members returned to Punjab to

S
agitate for rebellion alongside the Babbar Akali Movement.

IA
In 1915 they conducted revolutionary activities in central Punjab and organised
uprisings. Their presence shook the hold of the British empire and police
surveillance in Punjabi villages increased in an attempt to crush the rebellion.

Q Source:Chapter 12: Bipin Chandra: India's Struggle for Independence


TS
12 Consider the following statements.
1. The Constitution does not contain any specific procedure for the selection and
appointment of the Prime Minister.
2. The Constitution does not require that a person must prove his majority in the Lok
H
Sabha before he is appointed as the Prime Minister.
Which of the above is/are correct?
A. 1 only
G

B. 2 only
C. Both 1 and 2
D. None
SI

User Answer : C
Correct Answer : C
Answer Justification :
IN

Justification:Statement 1: Article 75 says only that the Prime Minister shall be


appointed by the president. However, this does not imply that the president is free to
appoint any one as the Prime Minister.

In accordance with the conventions of the parliamentary system of government, the


President has to appoint the leader of the majority party in the Lok Sabha as the
Prime Minister.

Statement 2: In 1980, the Delhi High Court held that the Constitution does not
require that a person must prove his majority in the Lok Sabha before he is
appointed as the Prime Minister. The President may first appoint him the Prime

(C) Insights Active Learning. | All rights reserved. www.insightsias.com 8


User Name : chandan paswan
Total Marks : 200
TEST - 16 Mark Scored : 104.67

Minister and then ask him to prove his majority in the Lok Sabha within a
reasonable period.

Q Source:Chapter 19: Indian Polity: M Laxmikanth

13 In his oath of secrecy, the Prime Minister swears that


1. All government proceedings will be kept secret.

S
2. No matter handled by the Prime Minister's Office will be disclosed to other
ministries.
Which of the above is/are correct?

IA
A. 1 only
B. 2 only
C. Both 1 and 2
D. None
User Answer : A
Correct Answer : D
TS
Answer Justification :

Justification:The PM swears that he will not directly or indirectly communicate or


reveal to any person or persons any matter which shall be brought under his
H
consideration or shall become known to his as prime minister for the Union except
as may be required for the due discharge of his duties as such Minister.
G

This does not imply that all government proceedings will be secret or Pm's office
will work in confidentiality. So, both statements are incorrect.
SI

Q Source: Chapter 19: Indian Polity: M Laxmikanth

14 Consider the following about Earth Hour.


1. It is organized by the World Wide Fund for Nature (WWF).
IN

2. The day organizes and encourages planting of at least one tree by every human
being at a specific time.
3. For India, the country-specific theme for 2016 was to encourage usage of solar
energy.
Select the correct answer using the codes below.
A. 1 and 2 only
B. 2 and 3 only
C. 1 and 3 only
D. 1, 2 and 3
User Answer :

(C) Insights Active Learning. | All rights reserved. www.insightsias.com 9


User Name : chandan paswan
Total Marks 200
TEST - 16 Mark Scored 104.67

Correct Answer : C
Answer Justification :

Learning:Earth Hour 2016 encouraged people to explore and adopt a cleaner and
more sustainable lifestyle by embracing renewable energy solutions.

This day encourages citizens, communities, businesses and organisations to switch


the lights off from 8:30 pm local time for an hour to highlight the plight of the

S
planet. The first Earth Hour was held in 2007 in Sydney, Australia.

IA
Q
Source:http://www.thehindu.com/news/cities/Hyderabad/residents-look-the-other-w
ay-during-earth-hour/article8376534.ece

15 Exchange-Traded Fund (ETF) is


TS
A. A basket of blue-chip company stocks
B. A basket of assets traded on stock market
C. FII that is received in India via the security markets
D. FDI in stock market companies operating in India
User Answer :
H
Correct Answer : B
Answer Justification :
G

Learning:The link explains the concepts very well

http://www.etf.com/etf-education-center/21011-what-is-an-etf.html
SI

An exchange-traded fund (ETF) is an investment fund traded on stock exchanges,


much like stocks.
IN

An ETF holds assets such as stocks, commodities, or bonds, and trades close to its
net asset value over the course of the trading day. Most ETFs track an index, such as
a stock index or bond index.

Q Source: Chapter 11: Ramesh Singh: Indian Economy

16 Consider the following statements.


1. The President may require the Council of Ministers to reconsider advice tendered by
it.
2. The advice tendered by Ministers to the President shall not be inquired into in any

(C) Insights Active Learning. | All rights reserved. www.insightsias.com 10


User Name : chandan paswan
Total Marks : 200
TEST - 16 Mark Scored : 104.67

court.
Which of the above is/are correct?
A. 1 only
B. 2 only
C. Both 1 and 2
D. None
User Answer : C

S
Correct Answer : C
Answer Justification :

IA
Justification: As per Article 74'Council of Ministers to aid and advise President.

There shall be a Council of Ministers with the Prime Minister at the head to aid and
advise the President who shall, in the exercise of his functions, act in accordance
with such advice.
TS
However, the President may require the Council of Ministers to reconsider such
advice and the President shall act in accordance with the advice tendered after such
reconsideration.

Q Source:Chapter 20: Indian Polity: M Laxmikanth


H

17 In 1971, the Supreme Court held that even after the dissolution of the Lok Sabha, the
G

council of ministers does not immediately cease to hold office. How does it help the
political system and administration in India?
A. It prevents the bureaucracy from usurping political power.
SI

B. It ensures the accountability of the executive to the legislature.


C. The Council of Ministers get an opportunity to again prove their party
majority on the house floor.
D. The president cannot exercise the executive power without the aid and advice
IN

of the council of ministers, thus it helps him.


User Answer : D
Correct Answer : D
Answer Justification :

Justification:Option (a): This option may seem correct as in absence of council of


Ministers, bureaucracy becomes an important decision-making authority. But, such
extreme generalizations should not be chosen as the answer. (d) is more appropriate.

Option (b): Once the house is dissolved, the very idea of accountability to the
legislature dissolves for the time being. Only the President then issues orders.

(C) Insights Active Learning. | All rights reserved. www.insightsias.com 11


User Name : chandan paswan
Total Marks 200
TEST - 16 Mark Scored 104.67

Option (c): Once the house is dissolved, the CoM cannot prove their majority again.
So, it is wrong.

Option (d): Article 74 is mandatory and, therefore, the president cannot exercise the
executive power without the aid and advise of the council of ministers. Any exercise
of executive power without the aid and advice will be unconstitutional as being
violative of Article 74'.

S
Q Source:Chapter 20: Indian Polity: M Laxmikanth

IA
18 Consider the following about Cabinet Committees.
1. They are extra-constitutional bodies.
2. They are only temporary in nature.
3. Non-cabinet ministers cannot be its members.
4. Only the Prime Minister is authorized to chair cabinet committees.
TS
Select the correct answer using the codes below.
A. 1 and 2 only
B. 3 and 4 only
C. 1 only
D. 2 and 4 only
H
User Answer : B
Correct Answer : C
Answer Justification :
G

Justification: Statement 1: They are not mentioned in the Constitution. However,


the Rules of Business provide for their establishment
SI

Statement 2: They are of two types'standing and ad hoc. The former are of a
permanent nature while the latter are of a temporary nature. So, 2 is wrong.
IN

Statement 3: They usually include only Cabinet Ministers. However, the non-cabinet
Ministers are not debarred from their membership.

Statement 4: They are mostly headed by the Prime Minister. Sometimes other
Cabinet Ministers, particularly the Home Minister or the Finance Minister, also acts
as their Chairman. But, in case the Prime Minister is a member of a committee, he
invariably presides over it.

Learning:They are set up by the Prime Minister according to the exigencies of the
time and requirements of the situation.They not only sort out issues and formulate
proposals for the consideration of the Cabinet, but also take decisions. However, the

(C) Insights Active Learning. | All rights reserved. www.insightsias.com 12


User Name : chandan paswan
Total Marks : 200
TEST - 16 Mark Scored : 104.67

Cabinet can review their decisions.

Q Source: Chapter 21: Indian Polity: M Laxmikanth

19 World Network of Biosphere Reserves includes which of the following protected areas
of India?
1. Gulf of Mannar

S
2. Sundarban
3. Namdapha
4. Kanchenjunga

IA
5. Agasthyamala
Select the correct answer using the codes below.
A. 1, 2 and 5 only
B. 2, 3 and 4 only
C. 1, 3 and 5 only
D. 1, 2, 3, 4 and 5
TS
User Answer : A
Correct Answer : A
Answer Justification :
H
Learning:India's Agasthyamala Biosphere Reserve (ABR) was included in the
World Network of Biosphere Reserves of United Nations Educational, Scientific
and Cultural Organization (UNESCO).
G

With the addition of the ABR, total 10 of the 18 biosphere reserves in the country
have made it to the list of World Network of Biosphere Reserves.
SI

The other 9 Biosphere Reserves are: Nilgiri (Western Ghats), Gulf of Mannar
(Tamil Nadu), Sunderban (West Bengal), Nanda Devi (Uttarakhand), Nokrek
(Meghalaya), Pachmarh (Madhya Pradesh), Similipal (Odisha), Achanakmar-
IN

Amarkantak (Madhya Pradesh and Chhattisgarh) and Great Nicobar.

Q
Source:http://www.thehindu.com/news/national/kerala/indias-agasthyamala-among-
20-unesco-world-biosphere-reserves/article8381525.ece

20 Consider the following statements.


1. Assertion (A): Though the President of India is not a member of either House of
Parliament, he is an integral part of the institution of Parliament.
2. Reason (R): A bill passed by both the Houses of Parliament cannot become law

(C) Insights Active Learning. | All rights reserved. www.insightsias.com 13


User Name : chandan paswan
Total Marks 200
TEST - 16 Mark Scored 104.67

without the President's assent.


In the context of the above, which of these is correct?
A. A is correct, and R is an appropriate explanation of A.
B. A is correct, but R is not an appropriate explanation of A.
C. A is correct, but R is incorrect.
D. A is incorrect, but R is correct.
User Answer : A

S
Correct Answer : A
Answer Justification :

IA
Justification: He does not even sit in Parliamentary proceedings.

However, he also performs certain functions relating to the proceedings of the


Parliament, for example, he summons and prorogues both the Houses, dissolves the
Lok Sabha, addresses both the Houses, issues ordinances when they are not in
session, and so on.
TS
Q Source:Chapter 22: Indian Polity: M Laxmikanth

21 The parliamentary form of government emphasises on the


H
A. Separation of powers between the legislature and executive
B. System of Universal Adult Franchise
C. Interdependence between the legislative and executive organs
G

D. Democratic election system administered by an independent body


User Answer : D
Correct Answer : C
SI

Answer Justification :

Justification:option (a) is valid in the Presidential system as in USA.


IN

A Parliament may not be based on democratic elections or even adult franchise. It


can be an entirely nominated body as it largely used to be in the colonial rule. So,
(b) and (d) are incorrect.

In India, executive is a part of the legislature, and the legislature is led by the
executive. So, (c) is the most appropriate option.

Q Source:Chapter 22: Indian Polity: M Laxmikanth

22 Consider the following statements about Rajya Sabha.

(C) Insights Active Learning. | All rights reserved. www.insightsias.com 14


User Name : chandan paswan
Total Marks : 200
TEST - 16 Mark Scored : 104.67

1. The representatives of states in the Rajya Sabha are elected by the elected members
of state legislative assemblies.
2. The election to Rajya Sabha is held in accordance with the system of proportional
representation by means of the single transferable vote.
3. The seats are allotted to the states in the Rajya Sabha on the basis of population.
Select the correct answer using the codes below.
A. 1 and 2 only

S
B. 2 and 3 only
C. 1 and 3 only
D. 1, 2 and 3 only

IA
User Answer : D
Correct Answer : D
Answer Justification :

Learning:The seats are allotted to the states in the Rajya Sabha on the basis of
TS
population. Hence, the number of representatives varies from state to state.

For example, Uttar Pradesh has 31 members while Tripura has 1 member only.

However, in USA, all states are given equal representation in the Senate irrespective
of their population. USA has 50 states and the Senate has 100 members'2 from each
H
state. The representatives of each union territory in the Rajya Sabha are indirectly
elected by members of an electroral college specially constituted for the purpose.
G

Q Source: Chapter 22: Indian Polity: M Laxmikanth


SI

23 The Union Cabinet has given its approval for India to accede to the Ashgabat
Agreement. It establishes international transport and transit corridor between
A. Central Asia and the Persian Gulf countries
B. South-east Asia to European Countries
IN

C. West-African countries to South Asian countries


D. Oil producing Gulf countries and littoral states of Indian Ocean
User Answer : A
Correct Answer : A
Answer Justification :

Learning:It is a transit agreement established in year 2011.

Its founding members are Uzbekistan, Iran, Turkmenistan and Oman. Kazakhstan
had joined the grouping in 2015.

(C) Insights Active Learning. | All rights reserved. www.insightsias.com 15


User Name : chandan paswan
Total Marks 200
TEST - 16 Mark Scored 104.67

Accession to the Agreement would enable India to:

Utilise this existing transport and transit corridor to facilitate trade and
commercial interaction and ties with the Eurasian region.
Synchronise India's efforts to implement the International North South
Transport Corridor (INSTC) for enhanced connectivity.
It will provide India an opportunity for reorientation of the freight traffic

S
from the traditional sea route to land transcontinental routes.

IA
Q Source:http://pib.nic.in/newsite/PrintRelease.aspx?relid=138309

24 The Constitution has empowered the Parliament to prescribe the manner of choosing the
representatives of the union territories in the Lok Sabha. Members of Lok Sabha from the
Union Territories are
TS
A. Chosen by direct election
B. Nominated by the Administrator/Lieutenant Governor of the UT
C. Recommended by the Union Cabinet and chosen by the President
D. Chosen by indirect elections by the local bodies of the UTs
User Answer : A
H
Correct Answer : A
Answer Justification :
G

Learning:Accordingly, the Parliament has enacted the Union Territories (Direct


Election to the House of the People) Act, 1965, by which the members of Lok Sabha
from the union territories are also chosen by direct election.
SI

The representatives of each union territory in the Rajya Sabha are indirectly elected
by members of an electroral college specially constituted for the purpose.
IN

This election is also held in accordance with the system of proportional


representation by means of the single transferable vote. Out of the seven union
territories, only two (Delhi and Puducherry) have representation in Rajya Sabha.
The populations of other five union territories are too small to have any
representative in the Rajya Sabha.

Q Source:Chapter 22: Indian Polity: M Laxmikanth

25 Consider the following about the Delimitation Commission of India.


1. It is a statutory body.

(C) Insights Active Learning. | All rights reserved. www.insightsias.com 16


User Name : chandan paswan
Total Marks : 200
TEST - 16 Mark Scored : 104.67

2. It redraws the boundaries of both assembly and Lok Sabha constituencies.


3. Its orders cannot be challenged in a court of law.
4. The Lok Sabha cannot modify its orders.
5. It is setup every five years.
Select the correct answer using the codes below.
A. 1, 2 and 5 only
B. 3, 4 and 5 only

S
C. 1, 2, 3 and 4 only
D. 1, 4 and 5 only
User Answer : C

IA
Correct Answer : C
Answer Justification :

Justification: Statement 1: It is established by Government of India under the


provisions of the Delimitation Commission Act.
TS
Statement 2: The main task of the commission is to redraw the boundaries of the
various assembly and Lok Sabha constituencies based on a recent census.

The representation from each state is not changed during this exercise. However, the
numbers of SC and ST seats in a state are changed in accordance with the census.
H

Statement 3 and 4: The Commission is a powerful body whose orders cannot be


challenged in a court of law. The orders are laid before the Lok Sabha and the
G

respective State Legislative Assemblies. However, modifications are not permitted.

Statement 5: Delimitation commissions have been set up four times in the past - In
SI

1952, 1963, 1973 and 2002 under Delimitation Commission acts of 1952, 1962,
1972 and 2002.

The present delimitation of parliamentary constituencies has been done on the basis
IN

of 2001 census figures under the provisions of Delimitation Act, 2002.

However, the Constitution of India was specifically amended in 2002 not to have
delimitation of constituencies till the first census after 2026.

Thus, the present Constituencies carved out on the basis of 2001 census shall
continue to be in operation till the first census after 2026.

Q Source:Chapter 22: Indian Polity: M Laxmikanth

(C) Insights Active Learning. | All rights reserved. www.insightsias.com 17


User Name : chandan paswan
Total Marks 200
TEST - 16 Mark Scored 104.67

26 Sevottam is related to
A. Citizen's charter in public organizations
B. Grievance redressal programmes within the most vulnerable communities
C. Reform of Public Sector Banks (PSBs)
D. Providing support to first generation entrepreneurs under 'Make in India'
programmes
User Answer :

S
Correct Answer : A
Answer Justification :

IA
Learning:The Department of Administrative Reforms & Public Grievances
(DARPG), Ministry of Personnel, Public Grievances and Pensions has taken steps to
put in place Sevottam Compliant Citizen's Charter and Grievance Redress
Mechanism. TS
The Sevottam framework was designed by DARPG in 2006 as an assessment
improvement framework for public service delivery.

Q Source: Chapter 8: India Yearbook 2016


H
27 Consider the following about Rajya Sabha.
1. It is a permanent body and not subject to dissolution.
2. The retiring members of Rajya Sabha are eligible for re-election and re-nomination
G

any number of times.


3. The Constitution fixes the term of the members of the Rajya Sabha.
4. The President can curtail the term of a Rajya Sabha member any time based on the
SI

recommendation of the Union cabinet.


Select the correct answer using the codes below.
A. 2, 3 and 4 only
B. 1 and 4 only
IN

C. 1 and 3 only
D. 1 and 2 only
User Answer : D
Correct Answer : D
Answer Justification :

Justification:Statement 1: It is a continuing chamber, that is, it is a permanent body


and not subject to dissolution. However, one-third of its members retire every
second year.

Statement 2: The seats left are filled up by fresh elections and presidential

(C) Insights Active Learning. | All rights reserved. www.insightsias.com 18


User Name : chandan paswan
Total Marks : 200
TEST - 16 Mark Scored : 104.67

nominations at the beginning of every third year.

The retiring members are eligible for re-election and re-nomination any number of
times.

Statement 3: The Constitution has not fixed the term of office of members of the
Rajya Sabha and left it to the Parliament. Accordingly, the Parliament in the
Representation of the People Act (1951) provided that the term of office of a

S
member of the Rajya Sabha shall be six years

IA
Statement 4: The act also empowered the president of India to curtail the term of
members chosen in the first Rajya Sabha. But, he cannot do it anytime based on any
recommendations. The term of MPs is fixed.

In the first batch, it was decided by lottery as to who should retire. Further, the act
also authorised the President to make provisions to govern the order of retirement of
TS
the members of the Rajya Sabha.

Q Source:Chapter 22: Indian Polity: M Laxmikanth


H
28 Which of the following is/are recognized as classical dance forms in India?
1. Odissi
2. Manipuri
G

3. Sattriya
4. Mohiniyattam
Select the correct answer using the codes below.
SI

A. 2, 3 and 4 only
B. 1, 2 and 3 only
C. 3 and 4 only
D. 1, 2, 3 and 4
IN

User Answer : B
Correct Answer : D
Answer Justification :

Learning:Dance in India has an unbroken tradition of over 2,000 years. Its themes
are derived from mythology, legends and classical literature, two main divisions
being classical and folk.

Classical dance forms are based on ancient dance discipline and have rigid
rules of presentation. Important among them are Bharata Natyam, Kathakali,

(C) Insights Active Learning. | All rights reserved. www.insightsias.com 19


User Name : chandan paswan
Total Marks 200
TEST - 16 Mark Scored 104.67

Kathak, Manipuri, Kuchipudi and Odissi.


Both classical and folk dances owe their present popularity to institutions like
Sangeet Natak Akademi and other training institutes and cultural
organizations.
The Akademi gives financial assistance to cultural institutions and awards
fellowships to scholars, performers and teachers to promote advanced study
and training in rare forms of dance and music.

S
Q Source: Chapter 8: India Yearbook 2016

IA
29 The governor can reserve a bill passed by the State Legislature for the consideration of
the President in which of the following cases?
1. If it is against the provisions of the Constitution.
2. If it is opposed to the Directive Principles of State Policy.
3. If it is against the larger interest of the country or of grave national importance.
TS
4. If it endangers the position of the State High Court
Select the correct answer using the codes below.
A. 1 and 4 only
B. 1, 2 and 3 only
C. 2, 3 and 4 only
H
D. 1, 2, 3 and 4
User Answer : A
Correct Answer : D
G

Answer Justification :

Justification:There is one more case when the bill can be reserved along with
SI

compulsory acquisition of property under Article 31A of the Constitution.

In case the bill passed by the state legislature endangers the position of the state high
court, such reservation is obligatory.
IN

Q Source: Chapter 26: Indian Polity: M Laxmikanth

30 The Speaker of the Lok Sabha derives his powers and duties from the Constitution of
India, the Rules of Procedure and Conduct of Business of Lok Sabha, and Parliamentary
Conventions. Accordingly, she exercises which of the following powers?
1. She is the final interpreter of the provisions of the Constitution of India for any
matter relating to the Parliament.
2. She has the final power in matters of decorum, order and business in the house.
3. She decides whether a bill is a money bill or not.

(C) Insights Active Learning. | All rights reserved. www.insightsias.com 20


User Name : chandan paswan
Total Marks : 200
TEST - 16 Mark Scored : 104.67

4. She appoints the chairman of all the parliamentary committees of the Lok Sabha and
supervises their functioning.
Select the correct answer using the codes below.
A. 2 and 3 only
B. 2, 3 and 4 only
C. 1, 2 and 3 only
D. 1 and 4 only

S
User Answer : C
Correct Answer : B
Answer Justification :

IA
Justification: Statement 1: Two things should be noted here:

a) She is the final interpreter of the entire constitution of India within the house
relating to its proceedings.
TS
b) She is the final interpreter only within the house. Outside the house, it is the
courts. So, 1 is wrong.

Statement 3: She decides whether a bill is a money bill or not and his decision on
this question is final.
H

When a money bill is transmitted to the Rajya Sabha for recommendation and
presented to the President for assent, the Speaker endorses on the bill his certificate
G

that it is a money bill.

Statement 4: She herself is the chairman of the Business Advisory Committee, the
SI

Rules Committee and the General Purpose Committee.

Q Source:Chapter 22: Indian Polity: M Laxmikanth


IN

31 The institutions of Speaker and Deputy Speaker originated in India in 1921 under
A. A Charter issued by the British crown for improving legislative business in
British India
B. Provisions of the Government of India Act of 1919 (Montague-Chelmsford
Reforms)
C. An order issued by the then Governor-General of India to manage the Central
Legislative Assembly better
D. Amendments made to the Charter Act of 1891 that created the office of
President and Vice-President of Legislative assemblies
User Answer : B

(C) Insights Active Learning. | All rights reserved. www.insightsias.com 21


User Name : chandan paswan
Total Marks 200
TEST - 16 Mark Scored 104.67

Correct Answer : B
Answer Justification :

Learning: At that time, the Speaker and the Deputy Speaker were called the
President and Deputy President respectively and the same nomenclature continued
till 1947.

Before 1921, the Governor- General of India used to preside over the meetings of

S
the Central Legislative Council.

IA
In 1921, the Frederick Whyte and Sachidanand Sinha were appointed by the
Governor-General of India as the first Speaker and the first Deputy Speaker
(respectively) of the central legislative assembly.

In 1925, Vithalbhai J. Patel became the first Indian and the first elected Speaker of
the central legislative assembly.
TS
The Government of India Act o 1935 changed the nomenclatures of President and
Deputy President of the Central Legislative Assembly to the Speaker and Deputy
Speaker respectively.
H
Q Source:Chapter 22: Indian Polity: M Laxmikanth
G

32 In reverse repos
A. The banks and financial institutions purchase Government securities from the
RBI
B. Banks and the financial institutions borrow money from the RBI for the
SI

short-term
C. RBI buys commercial papers from the public
D. None of the above
IN

User Answer : A
Correct Answer : A
Answer Justification :

Justification: Repos allow the banks and the financial institutions to borrow money
from the RBI for the short-term (by selling Government Securities to the RBI).

In reverse repos, the banks and financial institutions purchase Government securities
from the RBI (basically here the RBI is borrowing from the banks and the financial
institutions).

(C) Insights Active Learning. | All rights reserved. www.insightsias.com 22


User Name : chandan paswan
Total Marks : 200
TEST - 16 Mark Scored : 104.67

All the Government securities are dated and the interest for the repo or reverse repo
transactions is announced by the RBI from time to time.

The provision of repos and the reverse repos have been able to serve the liquidity
evenness in the economy as the banks are able to get the required amount of funds
out of it, and they can park surplus idle funds through it.

Q Source: Chapter 11: Ramesh Singh: Indian Economy

S
33 Each House of Parliament has separate secretarial staff of its own. The secretariat of

IA
each House is headed by a secretary-general. He is a permanent officer and is appointed by
the
A. Leader of the House
B. Presiding Officer of the House
C. President of India
TS
D. The Department of Personnel & Training
User Answer : B
Correct Answer : B
Answer Justification :
H
Learning: The Secretariat of Rajya Sabha was set up pursuant to the provisions
contained in Article 98 of the Constitution.
G

Parliament may by law regulate the recruitment and the conditions of service of
persons appointed to the secretarial staff of either House of Parliament.
SI

The President may, after consultation with the Speaker of the House of the People or
the Chairman of the Council of States, as the case may be, make rules regulating the
recruitment and the conditions of service of persons appointed to the secretarial staff
of the House
IN

Q Source:Chapter 22: Indian Polity: M Laxmikanth

34 Which of the following correctly point out the difference(s) between Adjournment and
Prorogation of a house?
1. Unlike Prorogation, Adjournment only terminates a sitting and not a session of the
House.
2. Unlike Adjournment, Prorogation leads to lapse of all the bills or any other business
pending before the House.
Select the correct answer using the codes below.

(C) Insights Active Learning. | All rights reserved. www.insightsias.com 23


User Name : chandan paswan
Total Marks 200
TEST - 16 Mark Scored 104.67

A. 1 only
B. 2 only
C. Both 1 and 2
D. None
User Answer : A
Correct Answer : A
Answer Justification :

S
Justification: Statement 1: Prorogation not only terminates a sitting but also a
session of the House.

IA
Statement 2: Prorogation does not affect the bills or any other business pending
before the House. This is the same as adjournment.

However, all pending notices (other than those for introducing bills) lapse on
prorogation and fresh notices have to be given for the next session.
TS
Q Source:Chapter 22: Indian Polity: M Laxmikanth

35 When the Lok Sabha is dissolved, all business including bills, motions, resolutions,
H
notices, petitions and so on pending before it or its committees lapse. However, certain
bills do not lapse on the dissolution of the Lok Sabha. They are?
1. A bill passed by the Lok Sabha but pending in the Rajya Sabha
G

2. A bill pending in the Rajya Sabha but not passed by the Lok Sabha
3. A bill passed by both Houses but pending assent of the president
Select the correct answer using the codes below.
SI

A. 1 and 2 only
B. 2 and 3 only
C. 1 and 3 only
D. 1, 2 and 3 only
IN

User Answer : C
Correct Answer : B
Answer Justification :

Learning:Some pending bills and all pending assurances that are to be examined by
the Committee on Government Assurances do not lapse on the dissolution of the
Lok Sabha. The position with respect to lapsing of bills is as follows:

A bill pending in the Lok Sabha lapses (whether originating in the Lok Sabha
or transmitted to it by the Rajya Sabha).

(C) Insights Active Learning. | All rights reserved. www.insightsias.com 24


User Name : chandan paswan
Total Marks : 200
TEST - 16 Mark Scored : 104.67

A bill passed by the Lok Sabha but pending in the Rajya Sabha lapses.
A bill not passed by the two Houses due to disagreement and if the president
has notified the holding of a joint sitting before the dissolution of Lok Sabha,
does not lapse.
A bill pending in the Rajya Sabha but not passed by the Lok Sabha does not
lapse.
A bill passed by both Houses but pending assent of the president does not

S
lapse.
A bill passed by both Houses but returned by the president for
reconsideration of Houses does not lapse.

IA
Q Source:Chapter 22: Indian Polity: M Laxmikanth

36 A starred question in Parliament, distinguished by an asterisk


A. Requires an oral answer and hence supplementary questions can follow
TS
B. Requires a written answer and hence supplementary questions can follow
C. Any question to which Ministers alone are answerable
D. One that is asked by giving a notice of less than ten days and answered orally
User Answer : A
Correct Answer : A
H
Answer Justification :

Justification: The first hour of every parliamentary sitting is slotted for the Question
G

hour.

During this time, the members ask questions and the ministers usually give answers.
SI

The questions are of three kinds, namely, starred, unstarred and short notice.

Option (b) is unstarred question.


IN

Option (d) is Short notice questions.

Q Source:Chapter 22: Indian Polity: M Laxmikanth

37 Which of the following statements about the Zero Hour in Parliament is INCORRECT?
A. The zero hour is not mentioned in the Rules of Procedure of the house.
B. Matters can be raised by members without prior notice.
C. The zero hour starts before the question hour after which the agenda for the
day is taken up.
D. It is an Indian innovation in the field of parliamentary procedures and has

(C) Insights Active Learning. | All rights reserved. www.insightsias.com 25


User Name : chandan paswan
Total Marks 200
TEST - 16 Mark Scored 104.67

been in existence since 1962.


User Answer : C
Correct Answer : C
Answer Justification :

Justification: Unlike the question hour, the zero hour is not mentioned in the Rules
of Procedure. Thus it is an informal device available to the members of the

S
Parliament to raise matters without any prior notice.

The zero hour starts immediately after the question hour and lasts until the agenda

IA
for the day (ie, regular business of the House) is taken up.

In other words, the time gap between the question hour and the agenda is known as
zero hour.

Q Source:Chapter 22: Indian Polity: M Laxmikanth


TS
38 Which of the following correctly point out the difference(s) between Censure Motion
and No Confidence Motion?
1. No-confidence motion should state the reasons for its adoption in the Lok Sabha,
H
Censure motion need not.
2. No-confidence motion can only be moved against the entire council of ministers,
Censure motion can be moved only against individual ministers.
G

Which of the above is/are correct?


A. 1 only
B. 2 only
SI

C. Both 1 and 2
D. None
User Answer : D
Correct Answer : D
IN

Answer Justification :

Justification:Statement 1: No-confidence motion need not state the reasons for its
adoption in the Lok Sabha, whereas Censure motion should.

Statement 2: Censure motion can be moved against an individual minister or a group


of ministers or the entire council of ministers. No-confidence motion can only be
moved against the entire council of ministers. So, 2 is wrong.

Learning: Censure motion is moved for censuring the council of ministers for
specific policies and actions. If it is passed in the Lok Sabha, the council of

(C) Insights Active Learning. | All rights reserved. www.insightsias.com 26


User Name : chandan paswan
Total Marks : 200
TEST - 16 Mark Scored : 104.67

ministers need not resign from the office. No-confidence motion is moved for
ascertaining the confidence of Lok Sabha in the council of ministers. If it is passed
in the Lok Sabha, the council of ministers must resign from office. The motion
needs the support of 50 members to be admitted.

Q Source:Chapter 22: Indian Polity: M Laxmikanth

S
39 The Financial Stability Board (FSB) that promotes international financial stability
through enhanced information exchange and international cooperation in financial market
supervision was established by

IA
A. The G-8 finance ministers and central bank governors
B. G-20 major economies
C. United Nations Economic and Social Council (UNECOSOC)
D. A resolution of all members of the UN General Assembly
User Answer : B
Correct Answer : B
TS
Answer Justification :

Learning: The Financial Stability Board (FSB) is an international body that


monitors and makes recommendations about the global financial system.
H
It was established after the 2009 G-20 London summit in April 2009 as a successor
to the Financial Stability Forum (FSF).
G

The Board includes all G-20 major economies, FSF members, and the European
Commission. It is based in Basel, Switzerland
SI

Q Source: Chapter 10: Ramesh Singh: Indian Economy

40 Which of the following statements about Public Bills is INCORRECT?


IN

1. It is introduced in the Parliament by a minister.


2. It can be introduced without any prior notice in the house.
Which of the above is/are correct?
A. 1 only
B. 2 only
C. Both 1 and 2
D. None
User Answer : B
Correct Answer : A
Answer Justification :

(C) Insights Active Learning. | All rights reserved. www.insightsias.com 27


User Name : chandan paswan
Total Marks 200
TEST - 16 Mark Scored 104.67

Justification: Statement 1: Public bills are introduced by Ministers, whereas private


bills are introduced by any Member of Parliament other than a minister.

Statement 2: Introduction of public bill in the House requires seven days' notice.
Introduction of private bill in the House requires one month's notice.

Learning: Public bill reflects of the policies of the government (ruling party). Its
rejection by the House amounts to the expression of want of parliamentary

S
confidence in the government and may lead to its resignation.

IA
Q Source:Chapter 22: Indian Polity: M Laxmikanth

41 Article 110 of the Constitution deals with the definition of money bills. It states that a
bill is deemed to be a money bill if it contains 'only' provisions dealing with all or any of
which of the following matters?
1. Abolition of a tax
TS
2. Regulation of the borrowing of money by the Union government
3. Appropriation of money out of the Consolidated Fund of India
Select the correct answer using the codes below.
A. 1 and 2 only
H
B. 2 and 3 only
C. 1 and 3 only
D. 1, 2 and 3
G

User Answer : D
Correct Answer : D
Answer Justification :
SI

Learning:It includes the following matters:

The imposition, abolition, remission, alteration or regulation of any tax;


IN

The regulation of the borrowing of money by the Union government;


The custody of the Consolidated Fund of India or the contingency fund of
India, the payment of moneys into or the withdrawal of money from any such
fund;
The appropriation of money out of the Consolidated Fund of India;
Declaration of any expenditure charged on the Consolidated Fund of India or
increasing the amount of any such expenditure;
The receipt of money on account of the Consolidated Fund of India or the
public account of India or the custody or issue of such money, or the audit of
the accounts of the Union or of a state; or
Any matter incidental to any of the matters specified above.

(C) Insights Active Learning. | All rights reserved. www.insightsias.com 28


User Name : chandan paswan
Total Marks : 200
TEST - 16 Mark Scored : 104.67

However, a bill is not to be deemed to be a money bill by reason only that it


provides for:

The imposition of fines or other pecuniary penalties, or


The demand or payment of fees for licenses or fees for services rendered; or
The imposition, abolition, remission, alteration or regulation of any tax by
any local authority or body for local purposes.

S
If any question arises whether a bill is a money bill or not, the decision of the
Speaker of the Lok Sabha is final.

IA
Q Source: Chapter 22: Indian Polity: M Laxmikanth

42 Advanced countries, in particular EU countries and Japan, have been witnessing a


decline in the energy intensity of GDP due to which of the following factors possibly?
1. Technological improvement
TS
2. Shift in the structure of their economies towards services
Which of the above is/are correct?
A. 1 only
B. 2 only
H
C. Both 1 and 2
D. None
User Answer :
G

Correct Answer : C
Answer Justification :
SI

Justification: Statement 1: Technological improvements reduce energy


consumption per unit of product generated, and there has been a phenomenal
expansion of energy saving technology, for e.g. LED bulbs replacing CFL bulbs.
IN

Statement 2: Services sector generally consumes less energy per capita as compared
to the industrial sector. So, the shift is more energy saving. Hence 2 is also correct.

Q Source: Chapter 10: Ramesh Singh: Indian Economy

43 A joint sitting to resolve the Parliamentary deadlock applies to which of the following
types of bills apart from ordinary bills?
1. Appropriation Bill as a part of Annual Budget
2. Constitutional Amendment Bills
3. Financial Bills

(C) Insights Active Learning. | All rights reserved. www.insightsias.com 29


User Name : chandan paswan
Total Marks 200
TEST - 16 Mark Scored 104.67

Select the correct answer using the codes below.


A. 1 and 2 only
B. 2 and 3 only
C. 3 only
D. 1 only
User Answer : C
Correct Answer : C

S
Answer Justification :

Justification: Joint sitting cannot also be called in case of a money bill.

IA
Statement 1: It is a money bill, and hence joint sitting cannot be summoned.

Statement 2: Each house has the same power with respect to a constitutional
amendment bill, and must pass it separately.
TS
Statement 3: In case of a disagreement between the two Houses over such a bill, the
President can summon a joint sitting of the two Houses to resolve the deadlock.

Q Source: Chapter 22: Indian Polity: M Laxmikanth


H
44 The expenditure 'charged' upon the Consolidated Fund of India
1. Cannot be discussed by Parliament
G

2. Is non-votable by the Rajya Sabha, but votable by Lok Sabha


3. Does not require Presidential assent
Select the correct answer using the codes below.
SI

A. 1 and 2 only
B. 1 only
C. 2 and 3 only
D. None of the above
IN

User Answer : A
Correct Answer : D
Answer Justification :

Justification: Statements 1and 2: The budget consists of two types of expenditure-


the expenditure 'charged' upon the Consolidated Fund of India and the expenditure
'made' from the Consolidated Fund of India.

The charged expenditure is non-votable by the Parliament, that is, it can only be
discussed by the Parliament, while the other type has to be voted by the Parliament.

(C) Insights Active Learning. | All rights reserved. www.insightsias.com 30


User Name : chandan paswan
Total Marks : 200
TEST - 16 Mark Scored : 104.67

Statement 3: The final bill containing all such expenditures must receive Presidential
assent to become valid.

Q Source: Chapter 22: Indian Polity: M Laxmikanth

45 The budget goes through six stages in the Parliament. Arrange the following in the
correct order of proceeding in Parliament.

S
1. Presentation of budget.
2. Scrutiny by departmental committees.
3. General discussion.

IA
4. Passing of finance bill
5. Passing of appropriation bill.
6. Voting on demands for grants.
Select the correct answer using the codes below.
A. 132564
B. 312654
TS
C. 312456
D. 132654
User Answer : D
Correct Answer : D
H
Answer Justification :

Learning: Correct order is:


G

Presentation of budget.
General discussion.
SI

Scrutiny by departmental committees


Voting on demands for grants.
Passing of appropriation bill.
Passing of finance bill.
IN

The general discussion on budget begins a few days after its presentation. It takes
place in both the Houses of Parliament and lasts usually for three to four days.

After the general discussion on the budget is over, the Houses are adjourned for
about three to four weeks and 24 departmental standing committees examine the
budget.

After voting demand for grants, an appropriation bill is introduced to provide for the
appropriation, out of the Consolidated Fund of India, all money required to meet:

(C) Insights Active Learning. | All rights reserved. www.insightsias.com 31


User Name : chandan paswan
Total Marks 200
TEST - 16 Mark Scored 104.67

The grants voted by the Lok Sabha.


The expenditure charged on the Consolidated Fund of India.

Q Source: Chapter 22: Indian Polity: M Laxmikanth

46 'Vote on account' provision is related to


A. Any grant made in advance by the Lok Sabha to the executive before the

S
passing of appropriation bill
B. Excess grant awarded by the Parliament to the executive that had been
appropriated in the annual budget

IA
C. Grant made before the 'March Rush' by the Lok Sabha to the executive
D. Grant sanctioned by Parliament to executive for meeting an unexpected
demand upon the resources of India
User Answer :
Correct Answer : A
Answer Justification :
TS
Learning: The Appropriation Bill becomes the Appropriation Act after it is
assented to by the President.
H
This act authorises (or legalises) the payments from the Consolidated Fund of India.
This means that the government cannot withdraw money from the Consolidated
Fund of India till the enactment of the appropriation bill. This takes time and usually
G

goes on till the end of April.

But the government needs money to carry on its normal activities after 31
SI

March (the end of the financial year).


To overcome this functional difficulty, the Constitution has authorised the
Lok Sabha to make any grant in advance in respect to the estimated
expenditure for a part of the financial year, pending the completion of the
IN

voting of the demands for grants and the enactment of the appropriation bill.
This provision is known as the 'vote on account'. It is passed (or granted) after
the general discussion on budget is over.
It is generally granted for two months for an amount equivalent to one-sixth
of the total estimation

Q Source: Chapter 22: Indian Polity: M Laxmikanth

47 Which of the following statements with reference to the Public Account of India is
CORRECT?

(C) Insights Active Learning. | All rights reserved. www.insightsias.com 32


User Name : chandan paswan
Total Marks : 200
TEST - 16 Mark Scored : 104.67

A. It is operated by executive action.


B. All taxes collected by the Government of India are submitted to this account.
C. It is used to fund contingencies and disaster management.
D. It is an extra-constitutional fund.
User Answer : A
Correct Answer : A
Answer Justification :

S
Justification & Learning: All other public money (other than those which are
credited to the Consolidated Fund of India) received by or on behalf of the

IA
Government of India shall be credited to the Public Account of India.

This includes provident fund deposits, judicial deposits, savings bank deposits,
departmental deposits, remittances and so on. This account is operated by executive
action, that is, the payments from this account can be made without parliamentary
appropriation.
TS
Such payments are mostly in the nature of banking transactions.

Q Source: Chapter 22: Indian Polity: M Laxmikanth


H
48 Which of the following is the authority to create or abolish the state legislative councils?
A. President of India
G

B. Union Council of Ministers


C. Parliament
D. Governor of the Concerned State
SI

User Answer : C
Correct Answer : C
Answer Justification :
IN

Learning: The question in hand shows the unitary tendencies of the Indian
federation cum Union.

The Parliament can create or abolish the state legislative councils on the
recommendation of the concerned state legislative assemblies.

It is notable that these councils are created by the Parliament too.

It can also increase or decrease the area, alter the boundaries and change the names
of states of the Indian Union.

(C) Insights Active Learning. | All rights reserved. www.insightsias.com 33


User Name : chandan paswan
Total Marks 200
TEST - 16 Mark Scored 104.67

Q Source: Chapter 22: Indian Polity: M Laxmikanth

49 In 2014, Supreme court of India setup a Committee under NR Madhava Menon to frame
guidelines for
A. Allowing new mining operations in left wing extremism affected areas
B. Government advertisements in print and media
C. Regulation of economic activities in Ecologically Sensitive Zones (ESZs)

S
D. Improving the socio-economic status of the Transgender
User Answer :
Correct Answer : B

IA
Answer Justification :

Learning: To keep politics away from such ads, the committee emphasized that
only the pictures and names of the President, the Prime Minister, Governor and
Chief Ministers should be published.
TS
50 Consider the following statements about a certain Parliamentary Committee.
1. The Rajya Sabha has no representation in this committee.
2. The committee examine the estimates included in the budget and suggest
'economies' in public expenditure.
H
3. The committee works throughout the financial year and reports to the House
as its examination proceeds.
The above refer to?
G

A. Committee on Public Undertakings


B. Departmental Standing Committees
C. Committee on Government Finances
SI

D. Estimates Committee
User Answer : D
Correct Answer : D
Answer Justification :
IN

Learning: All the thirty members are from Lok Sabha only.

These members are elected by the Lok Sabha every year from amongst its
members.

A minister cannot be elected as a member of the committee. The


chairman of the committee is appointed by the Speaker from amongst
its members and he is invariably from the ruling party.
The function of the committee is to examine the estimates included in
the budget and suggest 'economies' in public expenditure. Hence, it has

(C) Insights Active Learning. | All rights reserved. www.insightsias.com 34


User Name : chandan paswan
Total Marks : 200
TEST - 16 Mark Scored : 104.67

been described as a 'continuous economy committee'


It is not incumbent on the committee to examine the entire estimates of
any one year. The demands for grants are finally voted despite the fact
that the committee has made no report.

Q Source: Chapter 22: Indian Polity: M Laxmikanth

51 Consider the following statements.

S
1. Assertion (A): The single system of courts that enforces both Central laws as
well as the state laws has been adopted in India from the Government of India

IA
Act of 1935.
2. Reason (R): The Government of India Act of 1935 established the Federal
Court of India with original, appellate and advisory jurisdiction.
In the context of the above, which of these is correct?
A. A is correct, and R can be an appropriate explanation of A.
TS
B. A is correct, but R cannot be an appropriate explanation of A.
C. A is correct, but R is incorrect.
D. A is incorrect, but R is correct.
User Answer : D
Correct Answer : A
Answer Justification :
H

Justification: The Federal Court of India was a judicial body, established in


India in 1937 under the provisions of the Government of India Act 1935, with
G

original, appellate and advisory jurisdiction.

It functioned until 1950, when the Supreme Court of India was established.
SI

The Federal Court had exclusive original jurisdiction in any dispute between
the Central Government and the Provinces.
IN

Initially, it was empowered to hear appeals from the High Courts of the
provinces in the cases which involved the interpretation of any Section of the
Government of India Act, 1935.

Q Source: Chapter 25: Indian Polity: M Laxmikanth

52 A dispute between two or more states will fall under Supreme Court's
A. Exclusive Original Jurisdiction
B. Appellate Jurisdiction
C. Writ Jurisdiction

(C) Insights Active Learning. | All rights reserved. www.insightsias.com 35


User Name : chandan paswan
Total Marks 200
TEST - 16 Mark Scored 104.67

D. Advisory Jurisdiction
User Answer : A
Correct Answer : A
Answer Justification :

Learning: As a federal court, the Supreme Court decides the disputes


between different units of the Indian Federation. More elaborately, any

S
dispute between:

The Centre and one or more states; or

IA
The Centre and any state or states on one side and one or more states
on the other; or
Between two or more states.

In the above federal disputes, the Supreme Court has exclusive original
jurisdiction.
TS
Exclusive means, no other court can decide such disputes and original means,
the power to hear such disputes in the first instance, not by way of appeal.

Q Source: Chapter 25: Indian Polity: M Laxmikanth


H

53 The Supreme Court enjoys appellate jurisdiction in


1. Civil matters
G

2. Criminal matters
3. Constitutional matters
Select the correct answer using the codes below.
SI

A. 1 and 2 only
B. 2 and 3 only
C. 1 and 3 only
D. 1, 2 and 3
IN

User Answer : D
Correct Answer : D
Answer Justification :

Learning: The Supreme Court is primarily a court of appeal and hears


appeals against the judgements of the lower courts.

It enjoys a wide appellate jurisdiction which can be classified under four


heads:

Appeals in constitutional matters.

(C) Insights Active Learning. | All rights reserved. www.insightsias.com 36


User Name : chandan paswan
Total Marks : 200
TEST - 16 Mark Scored : 104.67

Appeals in civil matters.


Appeals in criminal matters.
Appeals by special leave.

In the constitutional cases, an appeal can be made to the Supreme Court


against the judgement of a high court if the high court certifies that the case
involves a substantial question of law that requires the interpretation of the
Constitution.

S
Q Source: Chapter 25: Indian Polity: M Laxmikanth

IA
54 The constitutional validity of a legislative enactment or an executive order can be
challenged in the Supreme Court on which of the following grounds?
1. If it infringes the Fundamental Rights
2. If it is repugnant to the constitutional provisions
3. If it is outside the competence of the authority which has framed it
TS
Select the correct answer using the codes below.
A. 1 and 2 only
B. 2 and 3 only
C. 1 and 3 only
D. 1, 2 and 3
H
User Answer : D
Correct Answer : D
Answer Justification :
G

Learning: Judicial review is the power of the Supreme Court to examine the
constitutionality of legislative enactments and executive orders of both the
SI

Central and state governments.

On examination, if they are found to be violative of the Constitution or


fundamental rights (ultra-vires), they can be declared as illegal,
IN

unconstitutional and invalid (null and void) by the Supreme Court.


Consequently, they cannot be enforced by the Government.

Judicial review is needed for the following reasons:

To uphold the principle of the supremacy of the Constitution.


To maintain federal equilibrium (balance between Centre and states).
To protect the fundamental rights of the citizens

Q Source: Chapter 25: Indian Polity: M Laxmikanth

(C) Insights Active Learning. | All rights reserved. www.insightsias.com 37


User Name : chandan paswan
Total Marks 200
TEST - 16 Mark Scored 104.67

55 How is the Governor of a state appointed?


A. Indirectly elected by the State Legislative Assembly
B. Nominated by the President
C. Elected by a collegium consisting of the heads of all local bodies of
the concerned State
D. Nominated by the State Government subject to the approval of the
Union Government

S
User Answer : B
Correct Answer : B
Answer Justification :

IA
Justification & Learning: The governor is neither directly elected by the
people nor indirectly elected by a specially constituted electoral college as is
the case with the president.
TS
He is appointed by the president by warrant under his hand and seal. In a
way, he is a nominee of the Central government. But, as held by the Supreme
Court in 1979, the office of governor of a state is not an employment under
the Central government.

It is an independent constitutional office and is not under the control of or


H
subordinate to the Central government.

Q Source: Chapter 26: Indian Polity: M Laxmikanth


G

56 Consider the following about the terms of the Office of the Governor.
1. The Governor can resign at any time by addressing a resignation letter to the
SI

Chief Minister of the State


2. The Constitution does not lay down the grounds upon which a governor may
be removed by the President.
Which of the above is/are correct?
IN

A. 1 only
B. 2 only
C. Both 1 and 2
D. None
User Answer : B
Correct Answer : B
Answer Justification :

Justification: Statement 1: The resignation letter is submitted to the


President, and not the Chief Minister. So, 1 is wrong

(C) Insights Active Learning. | All rights reserved. www.insightsias.com 38


User Name : chandan paswan
Total Marks : 200
TEST - 16 Mark Scored : 104.67

Statement 2: A governor holds office for a term of five years from the date on
which he enters upon his office. However, this term of five years is subject to
the pleasure of the President.

The Supreme Court held that the pleasure of the President is not justifiable.
The governor has no security of tenure and no fixed term of office. He may
be removed by the President at any time without mentioning any grounds for
his removal.

S
Q Source: Chapter 26: Indian Polity: M Laxmikanth

IA
57 Before entering upon his office, the President has to make and subscribe to an
oath or affirmation. Which of the following is/are part of the oath or affirmation
made by the President?
1. To preserve, protect and defend the Constitution and the law
2. To uphold the sovereignty and integrity of India
TS
3. Perform the duties of the office without fear or favour
Select the correct answer using the codes below.
A. 1 and 2 only
B. 2 and 3 only
C. 1 only
H
D. 1, 2 and 3
User Answer : D
Correct Answer : C
G

Answer Justification :

Justification: In his oath, the President swears:


SI

to faithfully execute the office;


to preserve, protect and defend the Constitution and the law; and
to devote himself to the service and well-being of the people of India.
IN

The oath of office to the President is administered by the Chief Justice of


India.

Oath in Statement 2 is taken by the judges of Supreme Court and High


Courts, and

Oath in Statement 2 is taken by the information commissioners, judges of


Supreme Court and High Courts etc

Q Source: Chapter 17: Indian Polity: M Laxmikanth

(C) Insights Active Learning. | All rights reserved. www.insightsias.com 39


User Name : chandan paswan
Total Marks 200
TEST - 16 Mark Scored 104.67

58 How is the Lok Sabha speaker chosen?


A. Nominated by a collegium of Prime Minister, Leader of Opposition
and Minister for Parliamentary affairs
B. Nominated by the party/coalition in majority in Lok Sabha
C. Elected by the house from a body of eminent citizens specially
selected for this purpose
D. Elected by the house from amongst its members

S
User Answer : D
Correct Answer : D
Answer Justification :

IA
Learning: The Speaker is elected by the Lok Sabha from amongst its
members (as soon as may be, after its first sitting).

Whenever the office of the Speaker falls vacant, the Lok Sabha elects another
TS
member to fill the vacancy. The date of election of the Speaker is fixed by the
President.

Usually, the Speaker remains in office during the life of the Lok Sabha.

Q Source: Chapter 22: Indian Polity: M Laxmikanth


H

59 Which of the following is/are the differences between the pardoning Powers of
President and Governor?
G

1. Governor cannot pardon a death sentence, President can.


2. Governor can pardon a sentence of court martial, President cannot
Which of the above is/are correct?
SI

A. 1 only
B. 2 only
C. Both 1 and 2
D. None
IN

User Answer : A
Correct Answer : A
Answer Justification :

Justification: Statement 1: Even if a state law prescribes for death sentence,


the power to grant pardon lies with the President and not the governor. But,
the governor can suspend, remit or commute a death sentence.

Statement 2: The President can grant pardon, reprieve, respite, suspension,


remission or commutation in respect to punishment or sentence by a court-

(C) Insights Active Learning. | All rights reserved. www.insightsias.com 40


User Name : chandan paswan
Total Marks : 200
TEST - 16 Mark Scored : 104.67

martial (military court). Governor cannot.

Q Source: Chapter 26: Indian Polity: M Laxmikanth

60 The Governor has constitutional discretion in which of the following cases?


1. Recommendation for the imposition of the President's Rule in the state
2. While exercising his functions as the administrator of an adjoining union
territory

S
3. Seeking information from the chief minister with regard to the administrative
and legislative matters of the state

IA
4. Reservation of a bill for the consideration of the President.
Select the correct answer using the codes below.
A. 1 and 2 only
B. 3 and 4 only
C. 1, 3 and 4 only
TS
D. 1, 2, 3 and 4
User Answer : D
Correct Answer : D
Answer Justification :

Learning: One other constitutional discretion is determining the amount


H
payable by the Government of Assam, Meghalaya, Tripura and Mizoram to
an autonomous Tribal District Council as royalty accruing from licenses for
mineral exploration.
G

In addition to the above constitutional discretion (i.e., the express discretion


mentioned in the Constitution), the governor, like the president, also has
SI

situational discretion (i.e., the hidden discretion derived from the exigencies
of a prevailing political situation) in the following cases:

Appointment of chief minister when no party has a clear-cut majority


IN

in the state legislative assembly or when the chief minister in office


dies suddenly and there is no obvious successor
Dismissal of the council of ministers when it cannot prove the
confidence of the state legislative assembly.
Dissolution of the state legislative assembly if the council of ministers
has lost its majority

It is to be noted that the President has no or little constitutional discretion.

Q Source: Chapter 26: Indian Polity: M Laxmikanth

(C) Insights Active Learning. | All rights reserved. www.insightsias.com 41


User Name : chandan paswan
Total Marks 200
TEST - 16 Mark Scored 104.67

61 The Union Commerce and Industry Ministry has notified foreign direct
investment (FDI) upto 49% in insurance and pension sector will be under automatic
route. What implication it will have for the sectors?
1. The Indian corporate laws would not apply to companies investing within the
49% FDI limit.
2. Foreign Companies investing in these sectors within the cap would not
require prior approval from government.

S
Which of the above is/are correct?
A. 1 only
B. 2 only

IA
C. Both 1 and 2
D. None
User Answer : B
Correct Answer : B
Answer Justification :
TS
Justification: An Indian company may receive Foreign Direct Investment
under the two routes as given under:

i. Automatic Route FDI is allowed under the automatic route without prior
approval either of the Government or the Reserve Bank of India in all
H
activities/sectors as specified in the consolidated FDI Policy, issued by the
Government of India from time to time.
G

ii. Government Route FDI in activities not covered under the automatic route
requires prior approval of the Government which are considered by the
Foreign Investment Promotion Board (FIPB), Department of Economic
SI

Affairs, Ministry of Finance.

So, clearly statement 2 is correct. Statement 1 is incorrect as corporate laws


will apply irrespective of the FDI coming in through automatic or
IN

government route.

Q Source:
http://www.dnaindia.com/money/report-dipp-notifies-49-fdi-under-automatic
-route-in-insurance-pension-sector-2193287

62 Consider the following about the ordinance-making Power of the President.


1. The ordinances have the same force and effect as an act of Parliament.
2. An ordinance can be issued even if only one House of Parliament is in
session.

(C) Insights Active Learning. | All rights reserved. www.insightsias.com 42


User Name : chandan paswan
Total Marks : 200
TEST - 16 Mark Scored : 104.67

3. It can be issued to amend the Constitution


4. It is subject to judicial review.
Select the correct answer using the codes below.
A. 1, 3 and 4 only
B. 1 only
C. 1, 2 and 4 only
D. 2 and 3 only

S
User Answer : C
Correct Answer : C
Answer Justification :

IA
Justification: Statement 1 and 3: An ordinance like any other legislation, can
be retrospective, that is, it may come into force from a back date. It may
modify or repeal any act of Parliament or another ordinance. It can alter or
amend a tax law also. However, it cannot be issued to amend the
Constitution.
TS
Statement 2: An ordinance can also be issued when only one House is in
session because a law can be passed by both the Houses and not by one
House alone. An ordinance made when both the Houses are in session is void.
H
Statement 3: He can make an ordinance only when he is satisfied that the
circumstances exist that render it necessary for him to take immediate action.
In Cooper case, (1970), the Supreme Court held that the President's
G

satisfaction can be questioned in a court on the ground of malafide.

Q Source: Chapter 17: Indian Polity: M Laxmikanth


SI

63 The president nominates 12 members to the Rajya Sabha from


A. The persons recommended by the National Integration Council
B. People who have special knowledge or practical experience in art,
IN

literature, science and social service


C. People who have contributed to immensely to Indian politics.
D. Eminent political scientists who have never contested an election
User Answer : B
Correct Answer : B
Answer Justification :

Learning: The rationale behind this principle of nomination is to provide


eminent persons a place in the Rajya Sabha without going through the
process of election.

(C) Insights Active Learning. | All rights reserved. www.insightsias.com 43


User Name : chandan paswan
Total Marks 200
TEST - 16 Mark Scored 104.67

Some of these members currently are Sachin Tendulkar, Anu Aga, K.


Prasaran, K.T.S. Tulsi.

It should be noted here that the American Senate has no nominated members.

Q Source: Chapter 22: Indian Polity: M Laxmikanth

64 Nepal has recently signed an agreement with China to build a strategic railway

S
link between the two countries through Tibet. What implication does it have for
India?

IA
1. It may reduce Nepal's dependency on Indian sea ports.
2. It may reduce India's dominance of Nepal's overall trade with Asia.
Which of the above is/are correct?
A. 1 only
B. 2 only
C. Both 1 and 2
TS
D. None
User Answer : C
Correct Answer : C
Answer Justification :
H
Learning: Presently, India has virtual monopoly in trade with the Nepal
which accounts for one third trade of the Himalayan landlocked country.
G

The agreements signed also include a feasibility study on the establishment of


a Free Trade Agreement (FTA) between both countries.
SI

China will also provide assistance to build a border bridge over the Simikot-
Hilsa road section that will connect Humla district of Nepal with Tibet.

The article in the Q Source explains the intricacies of the important issue
IN

well.

Q Source:
http://www.thehindu.com/news/international/nepal-inks-transit-treaty-with-ch
ina-to-have-first-rail-link/article8381195.ece

65 Part V of the Indian constitution deals with which of the following matters?
A. Organisation, privileges and powers of the Parliament
B. Special provisions relating to certain classes
C. Relations between the Union and the States
D. Provisions for acquisition and termination of citizenship

(C) Insights Active Learning. | All rights reserved. www.insightsias.com 44


User Name : chandan paswan
Total Marks : 200
TEST - 16 Mark Scored : 104.67

User Answer : A
Correct Answer : A
Answer Justification :

Learning: Articles 79 to 122 in Part V of the Constitution deal with the


organisation, composition, duration, officers, procedures, privileges, powers
and so on of the Parliament.

S
Under the Constitution, the Parliament of India consists of three parts viz, the
President, the Council of States and the House of the People.

IA
Though the President of India is not a member of either House of Parliament
and does not sit in the Parliament to attend its meetings, he is an integral part
of the Parliament.

Q Source: Chapter 22: Indian Polity: M Laxmikanth


TS
66 Consider the following statements about the role of the Prime Minister.
1. He allocates and reshuffles various portfolios among the ministers.
2. He can bring about the collapse of the council of ministers by resigning from
office.
H
Which of the above is/are correct?
A. 1 only
B. 2 only
G

C. Both 1 and 2
D. None
User Answer : C
SI

Correct Answer : C
Answer Justification :

Learning: Since the Prime Minister stands at the head of the council of
IN

ministers, the other ministers cannot function when the Prime Minister
resigns or dies.

In other words, the resignation or death of an incumbent Prime Minister


automatically dissolves the council of ministers and thereby generates a
vacuum

The resignation or death of any other minister, on the other hand, merely
creates a vacancy which the Prime Minister may or may not like to fill.

Q Source: Chapter 19: Indian Polity: M Laxmikanth

(C) Insights Active Learning. | All rights reserved. www.insightsias.com 45


User Name : chandan paswan
Total Marks 200
TEST - 16 Mark Scored 104.67

67 The Union Ministry of Health and Family Welfare has recently amended the
Schedule V of the Drugs and Cosmetics Rules, 1945. As per the new rules issued by
the Ministry
A. Clinical trials in India will be banned.
B. Prior government permission will be needed for clinical trials.
C. Clinical trials will be banned for academic purposes.
D. None of the above

S
User Answer : B
Correct Answer : D
Answer Justification :

IA
Justification: The new rules state that:

There is no need for repeat tests on animals for new drug registrations
if complete data from similar toxicity experiments already exists for
TS
drugs approved abroad.
However, no permission will be required to conduct clinical trial
intended for academic purposes in respect of approved drug
formulation

The amendments come after repeated appeals by People for the Ethical
H
Treatment of Animals (PETA) India and Union Women & Child
Development Minister Maneka Gandhi who is a well-known animal rights
activist.
G

Q Source:
http://www.newindianexpress.com/nation/Centre-Bans-Repeat-Drug-Tests-o
SI

n-Animals/2016/03/19/article3334598.ece

68 Which of the following statements about the Public Accounts Committee is


INCORRECT?
IN

A. Its members are nominated by the Speaker giving due representation


from all parties.
B. The term of office of the members is one year.
C. A minister cannot be chosen as a member of the committee.
D. The committee examines the annual audit reports of the comptroller
and auditor general of India (CAG).
User Answer : A
Correct Answer : A
Answer Justification :

(C) Insights Active Learning. | All rights reserved. www.insightsias.com 46


User Name : chandan paswan
Total Marks : 200
TEST - 16 Mark Scored : 104.67

Justification: At present, it consists of 22 members (15 from the Lok Sabha


and from the Rajya Sabha).

The members are elected by the Parliament every year from amongst its
members according to the principle of proportional representation by means
of the single transferable vote. Thus, all parties get due representation in it.

The chairman of the committee is appointed by the Speaker from amongst its

S
members.

IA
Until 1966-67, the chairman of the committee belonged tothe ruling party.
However, since 1967 a convention has developed whereby the chairman of
thecommittee is selected invariably from the Opposition

Q Source: Chapter 22: Indian Polity: M Laxmikanth


TS
69 'Drishti' deployed by the Indian Metrological Department helps
A. Provide real time visibility information to pilots
B. Provide quick weather forecasts to farmers
C. Detect early cyclones and tsunamis in coastal areas
D. Track the buildup of Monsoon in Northern region
H
User Answer :
Correct Answer : A
Answer Justification :
G

Learning: Generally due to the poor visibility, the aircraft operations would
often get disturbed that affects passenger traffic during winter and monsoon
SI

seasons especially in Northern India.

Drishti is a transmissometer that takes in account the real time weather


parameters like humidity, wind speed, temperature conditions, etc., along the
IN

runway in order to calculate the visibility distance offered at that particular


runway.

It is cost-effective mandatory system required at all airports as per


International Civil Aviation Organisation (ICAO) and World Meteorological
Organisation (WMO).

Q Source:
http://indianexpress.com/article/cities/mumbai/made-in-india-visibility-devic
e-to-be-installed-at-mumbai-airport-soon/

(C) Insights Active Learning. | All rights reserved. www.insightsias.com 47


User Name : chandan paswan
Total Marks 200
TEST - 16 Mark Scored 104.67

70 To be eligible for election as Vice-President, a person should fulfil which of the


following qualifications?
1. He should be qualified for election as a member of the Rajya Sabha.
2. He should not be affiliated to a political party.
3. He should not have ever held the office of the President.
Select the correct answer using the codes below.
A. 1 only

S
B. 2 and 3 only
C. 1 and 3 only
D. 1 and 2 only

IA
User Answer : A
Correct Answer : A
Answer Justification :

Justification: Statement 1: Article 102 of the Constitution lays down that a


TS
person shall be disqualified for being chosen as, and for being, a member of
either House of Parliament, e.g. unsound mind, undischarged insolvent etc.

So, the nominee to the office of the Vice-President must fulfil these
conditions apart from others.
H
Statement 2 and 3: He should not hold any office of profit under the Union
government or any state government or any local authority or any other
public authority. Conditions in 2 and 3 do not prevent him/her to file
G

nomination for election to the office of the Vice-President.

Even a sitting President or Vice-President of the Union, the governor of any


SI

state and a minister for the Union or any state is not deemed to hold any
office of profit and hence qualified for being a candidate for Vice-President.

Q Source: Chapter 18: Indian Polity: M Laxmikanth


IN

71 "Yugantar Ashram" is best known as the


A. First war memorial established by the British in respect of Indian
freedom fighters
B. Centre of Indian National Congress's agitations
C. Headquarters of the Gadar party
D. Hub of mutiny and revolutionary thought in India
User Answer : C
Correct Answer : C
Answer Justification :

(C) Insights Active Learning. | All rights reserved. www.insightsias.com 48


User Name : chandan paswan
Total Marks : 200
TEST - 16 Mark Scored : 104.67

Learning:The Hindustan Association of the Pacific Coast, known as the


Gadar Party was founded in 1913 to free India from British slavery. The
headquarters of the association was established initially at 436 Hill Street,
San Francisco and named as "Yugantar Ashram."

The GoI decided in 2013 to convert this memorial into a library and Museum.

http://www.sikhfoundation.org/people-events/gadar-memorial-in-san-francisc

S
o-to-be-museum-and-library/

IA
Q Source:Chapter 12: Bipin Chandra: India's Struggle for Independence

72 Kutiyattam is a form of
A. Folk art originating from the early Tamil country
B. Sanskrit theatre traditionally performed in the state of Kerala
C. Dance performed by female members of the Luriyum community of
TS
Andhra Pradesh
D. Poetic performance accompanied by a Mridangam in South India
User Answer : B
Correct Answer : B
Answer Justification :
H

Learning:It is officially recognised by UNESCO as a Masterpiece of the


Oral and Intangible Heritage of Humanity.
G

Traditionally, the main musical instruments used in Koodiyattam are


mizhavu, kuzhitalam, etakka, kurumkuzhal, and sankhu. Mizhavu, the most
SI

prominent of these, is a percussion instrument that is played by a person of


the Ambalavas Nambiar caste, accompanied by Nangyaramma playing the
kuzhithalam.
IN

Traditionally, Koodiyattam has been performed by Chakyars (a subcaste of


Kerala Hindus) and by Nangyaramma (women of the Ambalavasi Nambiar
caste).

Q Source: Chapter 5: India Yearbook 2016

73 The question of disqualification under the Anti-defection provisions under Tenth


Schedule of the constitution is decided by the
A. Chairman in the case of Rajya Sabha and Speaker in the case of Lok
Sabha
B. Supreme Court in case of Parliament and concerned High courts in

(C) Insights Active Learning. | All rights reserved. www.insightsias.com 49


User Name : chandan paswan
Total Marks 200
TEST - 16 Mark Scored 104.67

case of State legislative assemblies


C. President of India in all cases based on the recommendations of the
Union cabinet
D. Election Commission of India
User Answer : A
Correct Answer : A
Answer Justification :

S
Learning: The Constitution lays down that a person shall be disqualified from
being a member of Parliament if he is so disqualified on the ground of

IA
defection under the provisions of the Tenth Schedule. A member incurs
disqualification under the defection law:

If he voluntary gives up the membership of the political party on


whose ticket he is elected to the House;
TS
If he votes or abstains from voting in the House contrary to any
direction given by his political party;
If any independently elected member joins any political party; and
If any nominated member joins any political party after the expiry of
six months.
H
The question of disqualification under the Tenth Schedule is decided by the
Chairman in the case of Rajya Sabha and Speaker in the case of Lok Sabha
(and not by the president of India).
G

In 1992, the Supreme Court ruled that the decision of the Chairman/Speaker
in this regard is subject to judicial review.
SI

Q Source: Chapter 22: Indian Polity: M Laxmikanth

74 Which of the following fellowships are awarded by Sahitya Akademi?


IN

1. Sahitya Akademi Honorary Fellowhip


2. Anand Fellowship
3. Rabindranath Tagore Fellowship
Select the correct answer using the codes below.
A. 1 and 2 only
B. 2 and 3 only
C. 1 and 3 only
D. 1, 2 and 3
User Answer :
Correct Answer : A

(C) Insights Active Learning. | All rights reserved. www.insightsias.com 50


User Name : chandan paswan
Total Marks : 200
TEST - 16 Mark Scored : 104.67

Answer Justification :

Learning: The third fellowship (statement 3) is Premchand fellowship.

The Akademi confers its fellowship, its highest honour to literary figures.
Sahitya Adademi also instituted a fellowship named after Premchand during
his 125th Birth Anniversary in 2005 for scholars doing research on Indian
literature or to creative writers from the countries of the SAARC region other

S
than India.

IA
Every year since its inception in 1954, the Sahitya Akademi awards prizes to
the most outstanding books of literary merit published in any of the major
Indian languages recognized by the Akademi.

Q Source: Chapter 5: India Yearbook 2016


TS
75 Consider the following about the powers of the President of India.
1. He can appoint an inter-state council to promote Centre-state and inter-state
cooperation.
2. He directly administers the union territories through administrators appointed
by him.
H
3. He can declare an area as scheduled area and has powers with respect to the
administration of scheduled areas and tribal areas.
Select the correct answer using the codes below.
G

A. 1 and 2 only
B. 2 and 3 only
C. 1 and 3 only
SI

D. 1, 2 and 3
User Answer : D
Correct Answer : D
Answer Justification :
IN

Learning: Apart from the above functions, he also has the following powers:

He can seek any information relating to the administration of affairs of


the Union, and proposals for legislation from the prime minister.
He can require the Prime Minister to submit, for consideration of the
council of ministers, any matter on which a decision has been taken by
a minister but, which has not been considered by the council.
He can appoint a commission to investigate into the conditions of SCs,
STs and other backward classes.

(C) Insights Active Learning. | All rights reserved. www.insightsias.com 51


User Name : chandan paswan
Total Marks 200
TEST - 16 Mark Scored 104.67

Q Source: Chapter 17: Indian Polity: M Laxmikanth

76 Consider the following about the India Tourism Development Corporation


Limited (ITDC).
1. It is owned by the Government of India and functions under the Ministry of
Culture.
2. It can develop tourism infrastructure through the public-private partnership

S
(PPP) mode in the country.
Which of the above is/are correct?
A. 1 only

IA
B. 2 only
C. Both 1 and 2
D. None
User Answer : B
Correct Answer : B
Answer Justification :
TS
Justification: Statement 1: The India Tourism Development Corporation
Limited (ITDC) is an Hospitality, retail and Education company owned by
Government of India, under Ministry of Tourism.
H
Established in 1966, it owns over 17 properties under the Ashok Group of
Hotels brand, across India.
G

Statement 2: Indian Tourism Development Corporation Ltd (ITDC) has set


up a new joint venture (JV) with IL&FS to develop tourism infrastructure
through the public-private partnership (PPP) mode in the country.
SI

The equal JV will be called Ashok Tourism Infrastructure Development


(Ashok Infra).
IN

Q Source: Chapter 5: India Yearbook 2016

77 Consider the following about the trends of National Income in India.


1. India's GDP growth rate has always increased post-2004.
2. India's GDP has never shrunk since 1991 reforms.
Which of the above is/are correct?
A. 1 only
B. 2 only
C. Both 1 and 2
D. None

(C) Insights Active Learning. | All rights reserved. www.insightsias.com 52


User Name : chandan paswan
Total Marks : 200
TEST - 16 Mark Scored : 104.67

User Answer : D
Correct Answer : B
Answer Justification :

Justification: Statement 1: It had slowed down during world recession and


euro zone slowdown post-2008. So, 1 is incorrect.

Statement 2: The growth rate of India's GDP has always been positive even if

S
it fluctuated since 1991. So, GDP had never shrunk since 1991.

IA
You can see the trends here.

http://statisticstimes.com/economy/gdp-of-india.php

Q Source: Chapter 6: India Yearbook 2016


TS
78 Consider the following statements.Assertion (A): India is the largest exporter in
commercial services.Reason (R): Services sector contributes to more than 50% of
India's GDP.In the context of the above, which of these is correct?
A. A is correct, and R is an appropriate explanation of A.
B. A is correct, but R is not an appropriate explanation of A.
H
C. A is correct, but R is incorrect.
D. A is incorrect, but R is correct.
User Answer : A
G

Correct Answer : D
Answer Justification :
SI

Learning:India was the eighth largest exporter in commercial services with


3.2 per cent share of world exports and the tenth largest importer with 2.6 per
cent share of world imports in 2014. So, clearly A is wrong.
IN

Services sector forms around 65% id India's GDP. SO, R is correct.

Q Source: Chapter 6: India Yearbook 2016

79 Which of the following sectors is/are included in the Index of industrial


production (IIP)?
1. Mining
2. Electricity
3. Fertilizers
Select the correct answer using the codes below.
A. 1 and 2 only

(C) Insights Active Learning. | All rights reserved. www.insightsias.com 53


User Name : chandan paswan
Total Marks 200
TEST - 16 Mark Scored 104.67

B. 2 and 3 only
C. 1 and 3 only
D. 1, 2 and 3
User Answer : A
Correct Answer : D
Answer Justification :

S
Learning: The Index of Industrial Production (IIP) is an index for India
which details out the growth of various sectors in an economy such as
mining, electricity and manufacturing. It includes all 8 core industries

IA
including steel, fertilizers etc.

The all India IIP is a composite indicator that measures the short-term
changes in the volume of production of a basket of industrial products during
a given period with respect to that in a chosen base period.
TS
It is compiled and published monthly by the Central Statistical Organisation
(CSO) six weeks after the reference month ends.

Q Source:Chapter 6: India Yearbook 2016


H
80 The Union executive consists of the
1. President
2. Vice-President
G

3. Prime Minister
4. Council of Ministers
5. Attorney-General of India
SI

6. Central Administrative Tribunal


7. Central Vigilance Commission
Select the correct answer using the codes below.
A. 1, 2, 3, 4 and 5 only
IN

B. 1, 2, 5, 6 and 7 only
C. 3 and 4 only
D. 1, 2, 3, 4, 5, 6 and 7
User Answer : C
Correct Answer : A
Answer Justification :

Justification: Statement 6: CAT is tribunal wielding judicial powers, and


cannot be said to be a part of the Union executive.

(C) Insights Active Learning. | All rights reserved. www.insightsias.com 54


User Name : chandan paswan
Total Marks : 200
TEST - 16 Mark Scored : 104.67

Statement 7: CVC acts as a quasi-judicial body that inquires into cases of


corruption in the Union government.

Articles 52-78 (Part V) of the Constitution deal with the Union executive.

President heads the Union executive.

Q Source: Chapter 17: Indian Polity: M Laxmikanth

S
81 India has signed a free trade agreement (FTA) with which of these nations?
1. Sri Lanka

IA
2. China
3. European Union
Select the correct answer using the codes below.
A. 1 and 2 only
B. 1 only
TS
C. 1 and 3 only
D. 3 only
User Answer : B
Correct Answer : B
Answer Justification :
H

Learning: Such agreements involve cooperation between India and the


partner country to reduce trade barriers - import quotas and tariffs - and to
G

increase trade of goods and services with each other.

Here is the list


SI

http://commerce.nic.in/trade/international_ta.asp?id=2&trade=i

Q Source: Chapter 7: India Yearbook 2016


IN

82 The main objectives of the Special Economic Zone (SEZ) Act, 2005 are
1. Promotion of exports of goods and services
2. Promotion of investment from foreign sources
3. Development of infrastructure facilities
Select the correct answer using the codes below.
A. 1 and 2 only
B. 2 and 3 only
C. 1 and 3 only
D. 1, 2 and 3
User Answer : D

(C) Insights Active Learning. | All rights reserved. www.insightsias.com 55


User Name : chandan paswan
Total Marks 200
TEST - 16 Mark Scored 104.67

Correct Answer : D
Answer Justification :

Learning: Special Economic Zone (SEZ) is a specifically delineated duty-


free enclave and shall be deemed to be foreign territory for the purposes of
trade operations and duties and tariffs.

In order words, SEZ is a geographical region that has economic laws different

S
from a country's typical economic laws.

IA
Usually the goal is to increase foreign investments. SEZs have been
established in several countries, including China, India, Jordan, Poland,
Kazakhstan, Philippines and Russia. North Korea has also attempted this to a
degree.

Q Source:Chapter 7: India Yearbook 2016


TS
83 Consider the following about the impeachment of President of India.
1. The impeachment charges can be initiated by either House of Parliament.
2. The Parliament investigates the impeachment charges.
3. No President has so far been impeached.
H
Select the correct answer using the codes below.
A. 1 and 2 only
B. 2 and 3 only
G

C. 1 only
D. 1, 2 and 3
User Answer : D
SI

Correct Answer : D
Answer Justification :

Justification:Statement 1: Impeachment is a quasi-judicial procedure in the


IN

Parliament. These charges should be signed by one-fourth members of the


House (that framed the charges).

Statement 2: After the impeachment resolution is passed by a majority of


two-thirds of the total membership of that House, it is sent to the other House,
which should investigate the charges.

The President has the right to appear and to be represented at such


investigation.

If the other House also sustains the charges and passes the impeachment

(C) Insights Active Learning. | All rights reserved. www.insightsias.com 56


User Name : chandan paswan
Total Marks : 200
TEST - 16 Mark Scored : 104.67

resolution by a majority of two-thirds of the total membership, then the


President stands removed from his office from the date on which the bill is so
passed.

Q Source: Chapter 17: Indian Polity: M Laxmikanth

84 Project Arrow related with Service Quality Improvement is an initiative of


A. Department of Posts

S
B. Ministry of Defence
C. Ministry of Tourism

IA
D. Ministry of Tribal Welfare
User Answer : A
Correct Answer : A
Answer Justification :

Learning:Project Arrow entails comprehensive improvement of the 'core


TS
operations' of the Post Offices as well as modernized ambience under 'Look
& Feel' silo of the project.

Enhancing the quality of services in 'core areas' envisages focus on mail


delivery, remittances, savings bank and office service levels through
H
continuous monitoring of Key Performance Indicators, identifying problem
areas and take remedial action on a real-time basis, including making
systemic changes to improve service quality.
G

Q Source: Chapter 8: India Yearbook 2016


SI

85 The system of proportional representation aims at removing the defects of


territorial representation, where all sections of the people get representation in
proportion to their number. What types of proportional representation are followed
in India?
IN

1. List system
2. Single transferable vote system
3. Mixed member system
Select the correct answer using the codes below.
A. 1 and 2 only
B. 2 only
C. 1 and 3 only
D. 3 only
User Answer : B
Correct Answer : B

(C) Insights Active Learning. | All rights reserved. www.insightsias.com 57


User Name : chandan paswan
Total Marks 200
TEST - 16 Mark Scored 104.67

Answer Justification :

Learning: The simple majority system of representation does not represent


the whole electorate. On the other hand, even the smallest section of the
population gets its due share of representation in the legislature in the PR
system.

There are two kinds of proportional representation, namely, single

S
transferable vote system and list system.
In India, the first kind is adopted for the election of members to the

IA
Rajya Sabha and state legislative council and for electing the President
and the Vice-President.
Though some members of the Constituent Assembly had advocated
the system of proportional representation for the election of members
to the Lok Sabha, the Constitution has not adopted the system due to
TS
two reasons.
Difficulty for the voters to understand the system (which is
complicated) due to low literacy scale in the country.
Unsuitability to the parliamentary government due to the
tendency of the system to multiply political parties leading to
instability in government.
H

Q Source: Chapter 22: Indian Polity: M Laxmikanth


G

86 Which of the following categories of information are excluded from disclosure


under the Right to Information Act, 2005?
1. Information received in confidence from foreign Government
SI

2. Information which would impede the process of investigation of offenders


3. Cabinet papers including records of deliberations of the Council of Ministers
Select the correct answer using the codes below.
A. 1 and 2 only
IN

B. 2 and 3 only
C. 1 and 3 only
D. 1, 2 and 3
User Answer : A
Correct Answer : D
Answer Justification :

Learning: Other categories of information are:

Information, disclosure of which would prejudicially affect the

(C) Insights Active Learning. | All rights reserved. www.insightsias.com 58


User Name : chandan paswan
Total Marks : 200
TEST - 16 Mark Scored : 104.67

sovereignty and integrity of India, the security, "strategic, scientific or


economic" interests of the State, relation with foreign State or lead to
incitement of an offense;
Information which has been expressly forbidden to be published by
any court of law or tribunal or the disclosure of which may constitute
contempt of court;
Information, the disclosure of which would cause a breach of privilege

S
of Parliament or the State Legislature;
Information including commercial confidence, trade secrets or
intellectual property, the disclosure of which would harm the

IA
competitive position of a third party, unless the competent authority is
satisfied that larger public interest warrants the disclosure of such
information.

Q Source:Chapter 8: India Yearbook 2016


TS
87 Eurozone Crisis did NOT hit which of these countries?
1. Portugal
2. Spain
3. Italy
4. Ireland
H
Select the correct answer using the codes below.
A. 1 and 2 only
B. 3 and 4 only
G

C. 2 only
D. It hit all of them.
User Answer :
SI

Correct Answer : D
Answer Justification :

Learning:The eurozone, a currency union of 17 European countries, has


IN

been going through a major crisis which started with Greece but spread
rapidly to Ireland, Portugal, and Spain and subsequently Italy.

Sparked off by fear over the sovereign debt crisis in Greece, it went on to
impact the peripheral economies as well, especially those with over-leveraged
financial institutions. These economies, especially Greece, have witnessed
downgrades in the ratings of their sovereign debt due to fears of default and a
rise in borrowing costs.

Q Source: Chapter 10: Ramesh Singh: Indian Economy

(C) Insights Active Learning. | All rights reserved. www.insightsias.com 59


User Name : chandan paswan
Total Marks 200
TEST - 16 Mark Scored 104.67

88 Foreign exchange reserves are the foreign-currency deposits held by national


central banks and monetary authorities. Which of these countries has the highest
foreign exchange reserves?
A. Japan
B. China
C. USA
D. Russia

S
User Answer : B
Correct Answer : B
Answer Justification :

IA
Learning:China's forex is approximately $3 trillion, Japan's around 1.2
trillion. India's forex is around $0.35 trillion.

China maintains a large reserve to manage its exchange rate by selling and
TS
buying foreign currency in relation to the Chinese currency Yuan.

Q Source: Chapter 10: Ramesh Singh: Indian Economy

89 Consider the following statements.Assertion (A): India is characterized by a


relatively lower energy intensity of GDP as compared to emerging economies like
H
China and South Africa.Reason (R): Around80 per cent of the crude oil consumed in
India is imported.In the context of the above, which of these is correct?
Which of the above is/are correct?
G

A. A is correct, and R is an appropriate explanation of A.


B. A is correct, but R is not an appropriate explanation of A.
C. A is correct, but R is incorrect.
SI

D. Both A and R are incorrect.


User Answer : A
Correct Answer : B
Answer Justification :
IN

Justification: It is lower as compared to China, South Africa, and Russia but


higher than that of Brazil. So, A is correct.

R is also correct, but it does not explain statement A because:

Energy intensity of GDP is a structural characteristics of the economy.


The economy may import all its energy needs and yet have higher
energy intensity.
Crude oil is just one part of the energy basket.

(C) Insights Active Learning. | All rights reserved. www.insightsias.com 60


User Name : chandan paswan
Total Marks : 200
TEST - 16 Mark Scored : 104.67

Q Source:Chapter 10: Ramesh Singh: Indian Economy

90 Consider the following statements about a money bill.


1. A money bill can only be introduced in the Lok Sabha.
2. Rajya Sabha cannot reject or amend a money bill.
Which of the above is/are correct?
A. 1 only

S
B. 2 only
C. Both 1 and 2
D. None

IA
User Answer : C
Correct Answer : C
Answer Justification :

Justification: The Constitution lays down a special procedure for the passing
TS
of money bills in the Parliament.

A money bill can only be introduced in the Lok Sabha and that too on the
recommendation of the president. Every such bill is considered to be a
government bill and can be introduced only by a minister.
H
After a money bill is passed by the Lok Sabha, it is transmitted to the Rajya
Sabha for its consideration.
G

The Rajya Sabha has restricted powers with regard to a money bill. It cannot
reject or amend a money bill. It can only make the recommendations.
SI

It must return the bill to the Lok Sabha within 14 days, wither with or without
recommendations. The Lok Sabha can either accept or reject all or any of the
recommendations of the Rajya Sabha.
IN

Q Source:Chapter 22: Indian Polity: M Laxmikanth

91 The Indian President, head of the State, is not elected directly by the people of
India. He is elected by members of an electoral college consisting of
1. Former Presidents
2. Bharat Ratna Awardees
3. All Members of Parliament (MPs)
4. All Members of Legislative Assemblies of all states (MLAs)
5. All Members of Legislative Councils of states where applicable (MLCs)
Select the correct answer using the codes below.

(C) Insights Active Learning. | All rights reserved. www.insightsias.com 61


User Name : chandan paswan
Total Marks 200
TEST - 16 Mark Scored 104.67

A. 3 and 4 only
B. 1, 2 and 3 only
C. 3, 4 and 5 only
D. None of the above
User Answer : A
Correct Answer : D
Answer Justification :

S
Justification: Statement 1 and 2: They are not included in the collegium.

IA
Statement 3 and 4: Only elected members participate. Nominated members
do not. The rationale is that nominated members are chosen by the incumbent
President. If they participate in the election of the President, it confers an
unfair advantage to the incumbent.

Statement 5: MLCs do not participate in the election.


TS
The elected members of the legislative assemblies of the Union Territories of
Delhi and Puducherry also participate. Its nominated members do not
participate.
H
Q Source: Chapter 17: Indian Polity: M Laxmikanth

92 The Adjournment motion is introduced in the Parliament to draw attention of the


G

House to a definite matter of urgent public importance. Which of the following


statements about it is INCORRECT?
A. It needs the support of 50 members to be admitted.
SI

B. It involves an element of censure against the government.


C. Rajya Sabha is not permitted to make use of this device.
D. If it is passed by a special majority, the sitting of the house is
adjourned after the discussion.
IN

User Answer : A
Correct Answer : D
Answer Justification :

Justification: As it interrupts the normal business of the House, it is regarded


as an extraordinary device.

It involves an element of censure against the government and hence Rajya


Sabha is not permitted to make use of this device. The discussion on an
adjournment motion should last for not less than two hours and thirty

(C) Insights Active Learning. | All rights reserved. www.insightsias.com 62


User Name : chandan paswan
Total Marks : 200
TEST - 16 Mark Scored : 104.67

minutes. So, (b) and (c) are correct.

The house need not be adjourned. So, (d) is wrong.

Q Source: Chapter 22: Indian Polity: M Laxmikanth

93 Which of the following is/are part of the Organised Indian Money Market?
1. Mutual Funds

S
2. EXIM Bank
3. NABARD

IA
Select the correct answer using the codes below.
A. 1 only
B. 1 and 3 only
C. 1 and 2 only
D. 2 and 3 only
User Answer : A
TS
Correct Answer : A
Answer Justification :

Learning: There are around eight instruments or components of the Indian


money market especially designed to fulfill the
H

short-term fund requirements of the different categories of the individuals,


institutions or the firms and companies:
G

Treasury Bills
Call Money Market
SI

Certificate of Deposit
Commercial Bills
Commercial Papers
Mutual Funds
IN

Repo and Reverse Repo Markets.


Cash Management Bill

NABARD and EXIM bank are part of the financial markets.

Q Source: Chapter 11: Ramesh Singh: Indian Economy

94 Which of the following is/are Financial Regulators in India?


1. National Housing Board
2. SIDBI
3. NABARD

(C) Insights Active Learning. | All rights reserved. www.insightsias.com 63


User Name : chandan paswan
Total Marks 200
TEST - 16 Mark Scored 104.67

Select the correct answer using the codes below.


A. 3 only
B. 1 and 3 only
C. 1 and 2 only
D. 1, 2 and 3
User Answer :
Correct Answer : D

S
Answer Justification :

Learning: NABARD finances and regulates the micro-finance sector in

IA
India.

National Bank for Agriculture and Rural Development (NABARD), Small


Industries Development Bank of India (SIDBI), and National Housing Bank
(NHB) perform quasi-regulatory functions.
TS
NABARD supervises regional rural banks as well as state and district
cooperative banks.

NHB regulates housing finance companies, and SIDBI regulates the state
finance corporations.
H

Q Source: Chapter 11: Ramesh Singh: Indian Economy


G

95 The President appoints a member of the Lok Sabha as the Speaker Pro Tem when
the Speaker of the last Lok Sabha vacates her office. What are the duties of Speaker
Pro Tem.
SI

1. Administering oath of newly elected members


2. Enabling the house to select new Speaker
Which of the above is/are correct?
A. 1 only
IN

B. 2 only
C. Both 1 and 2
D. None
User Answer : C
Correct Answer : C
Answer Justification :

Learning:As provided by the Constitution, the Speaker of the last Lok Sabha
vacates his office immediately before the first meeting of the newly- elected
Lok Sabha.

(C) Insights Active Learning. | All rights reserved. www.insightsias.com 64


User Name : chandan paswan
Total Marks : 200
TEST - 16 Mark Scored : 104.67

Therefore, the President appoints a member of the Lok Sabha as the Speaker
Pro Tem.

Usually, the seniormost member is selected for this.


The President himself administers oath to the Speaker Pro Tem.
The Speaker Pro Tem has all the powers of the Speaker. He presides
over the first sitting of the newly-elected Lok Sabha.
His main duty is to administer oath to the new members.

S
He also enables the House to elect the new Speaker.
When the new Speaker is elected by the House, the office of the

IA
Speaker Pro Tem ceases to exist. Hence, this office is a temporary
office, existing for a few days

Q Source:Chapter 22: Indian Polity: M Laxmikanth

96 Under the Liquidity Adjustment Facility (LAF)


TS
A. RBI stands ready to lend to or borrow money from the banking
system, as per the latter's requirement at fixed interest rates
B. RBI manages the liquidity in the banking system by way of
sterilization
C. RBI sells government securities in the market to reduce the money
H
supply
D. RBI obliges the bank to categorise and prioritise their lending limits to
different sectors
G

User Answer :
Correct Answer : A
Answer Justification :
SI

Learning:The LAF is the key element in the monetary policy operating


framework of the RBI (introduced in 2000).
IN

On daily basis, the RBI stands ready to lend to or borrow money from the
banking system, as per the latter's requirement, at fixed interest rates.

The primary aim of such an operation is to assist banks to adjust to their day-
to-day mismatches in liquidity, via repo and reverse repo operations.

OMOs are conducted by the RBI via the sale/purchase of government


securities (G-Sec) to/from the market, which is option (c).

Q Source: Chapter 11: Ramesh Singh: Indian Economy

(C) Insights Active Learning. | All rights reserved. www.insightsias.com 65


User Name : chandan paswan
Total Marks 200
TEST - 16 Mark Scored 104.67

97 The Prime Minister is the leader of the Lower House. In this capacity, he enjoys
which of the following powers?
1. He advises the President with regard to summoning and proroguing of the
sessions of the Parliament.
2. He can recommend dissolution of the Lok Sabha to President at any time.
3. He has the authority to adjourn the lower house sine die.
Select the correct answer using the codes below.

S
A. 1 and 2 only
B. 2 and 3 only
C. 1 and 3 only

IA
D. 1, 2 and 3
User Answer : A
Correct Answer : A
Answer Justification :TS
Justification:Statement 2: Both the executive and legislature in India have
powers to shake each other's existence. Executive (PM) can recommend
dissolution of lower house to the President.

On the other hand, legislature can pass no-confidence motion to remove the
council of Ministers from office.
H

Statement 3: Speaker, Lok Sabha only is authorized to do so.


G

Q Source:Chapter 19: Indian Polity: M Laxmikanth

98 Non-Banking financial companies (NBFCs) differ from banks in


SI

1. An NBFC cannot run a savings account for an individual, unlike banks.


2. It cannot issue cheques to its customers, unlike banks.
3. No NBFC needs to get registered with RBI, unlike banks.
Select the correct answer using the codes below.
IN

A. 1 and 2 only
B. 2 and 3 only
C. 1 and 3 only
D. 1, 2 and 3
User Answer : D
Correct Answer : A
Answer Justification :

Justification: Statement 1: An NBFC cannot accept demand deposits (which


are payable on demand), like the savings and current accounts.

(C) Insights Active Learning. | All rights reserved. www.insightsias.com 66


User Name : chandan paswan
Total Marks : 200
TEST - 16 Mark Scored : 104.67

Statement 2: It is not a part of the payment and settlement system and as such
cannot issue cheques to its customers.

Statement 3: Under the RBI Act, 1934, the NBFCs have to get registered with
RBI. However, to obviate dual regulation, certain category of NBFCs which
are regulated by other regulators are exempted from the requirement of
registration with RBI.

S
Q Source: Chapter 11: Ramesh Singh: Indian Economy

IA
99 Consider the following about the Indian Home Rule movement.
1. It wanted to obtain the status of a Dominion for India within the British
Empire.
2. It worked as a joint movement of the Indian National Congress and the All
India Muslim League.
3. Gandhiji was once elected as a President of the All India Home Rule League.
TS
Select the correct answer using the codes below.
A. 1 and 2 only
B. 2 and 3 only
C. 1 and 3 only
D. 1, 2 and 3
H
User Answer : B
Correct Answer : C
Answer Justification :
G

Justification:Statement 1: Between 1916 and 1918, when the World War-I


was gradually approaching an end, prominent Indians like Muhammad Ali
SI

Jinnah, Bal Gangadhar Tilak, Sir S.

Subramania Iyer, Annie Besant decided to organise a national alliance of


leagues across India.
IN

It was specifically to demand Home Rule, or self-government within the


British Empire for all of India.

Statement 2: It was run separately from both parties. Some members of both
INC and AIML joined the Home Rule movement.

Statement 3: In 1920, the All India Home Rule League elected Mahatma
Gandhi as its President. In a year, the body would merge into the Indian
National Congress to form a united Indian political front.

(C) Insights Active Learning. | All rights reserved. www.insightsias.com 67


User Name : chandan paswan
Total Marks 200
TEST - 16 Mark Scored 104.67

Q Source:Chapter 13: Bipin Chandra: India's Struggle for Independence

100 Which of the following is/are the executive powers and functions of the
Governor?
1. All executive actions of the government of a state are formally taken in his
name.
2. He can make rules for more convenient transaction of the business of a state

S
government and for the allocation among the ministers of the said business.
3. He appoints the chief minister and other ministers.
4. He has the power to both appoint and remove the state election

IA
commissioner.
5. He acts as the chancellor of universities in the state.
6. He can impose Governor's rule in the State based on a report of State
Secretariat.
Select the correct answer using the codes below.
TS
A. 1, 2, 3, 4 and 5 only
B. 1, 2, 3 and 4 and 6 only
C. 4, 5 and 6 only
D. 1, 2, 3 and 5 only
User Answer : D
H
Correct Answer : D
Answer Justification :
G

Justification: Statement 1: He can make rules specifying the manner in


which the Orders and other instruments made and executed in his name shall
be authenticated.
SI

Statement 2 and 3: He not only allocated business, but also appoints the chief
minister and other ministers.

Statement 4: He appoints the state election commissioner and determines his


IN

conditions of service and tenure of office. However, the state election


commissioner can be removed only in like manner and on the like grounds as
a judge of a high court, and not by the Governor.

Statement 5: He also appoints the vice-chancellors of universities in the state.

Statement 6: In the state of Jammu and Kashmir, failure of constitutional


machinery results in Governor's rule, imposed by invoking Section 92 of
Constitution of Jammu and Kashmir.

(C) Insights Active Learning. | All rights reserved. www.insightsias.com 68


User Name : chandan paswan
Total Marks : 200
TEST - 16 Mark Scored : 104.67

But, a) the proclamation can be issued by the state's Governor after obtaining
the consent of the President of India; and b) Governor's rule is not applicable
to Indian states except J&K. So, this statement would be incorrect.

Q Source: Chapter 26: Indian Polity: M Laxmikanth

S
IA
TS
H
G
SI
IN

(C) Insights Active Learning. | All rights reserved. www.insightsias.com 69

S-ar putea să vă placă și